You are on page 1of 44

Professional Education 9. Which characterizes a constructivist teaching-learning process?

a. Conceptual interrelatedness
1. With R.A. 9155, to which body were all the functions, programs, b. Multiple perspectives
and activities of the Department of Education related to Sports c. Authentic assessment
competition transferred? d. Passive acceptance of information
a. Technical Education Services Department Authority 10. On what theory is the sequencing of instruction anchored?
b. Philippine Sports Commission a. Gagne’s hierarchical theory
c. National Commission for Culture and the Arts b. B.F. Skinner’s operant conditioning theory
d. Commission on Higher Education c. Bandura’s social learning theory
d. Thorndike’s law of effect
2. Parenting style influences children’s development. Read the 11. A common complaint of teachers about pupils is this: “You give
following parent’s remarks for their children then, answer the them assignment, the following day they come without any. You
question. teach them this today, asks them tomorrow and they don’t know. It
Parent C – Tells her child: “You should do it my way or else. There is as if there is nothing that you taught them at all.” Based on the
is no discussion.” theory of information processing, what must teachers do to
Parent D – Tells her husband: “It is 10:00 PM, do you know where counteract pupil’s forgetting?
your child is?” I. Punish every child who can’t give correct answers to
Parent E – Tells her child: “You know, you should have not done questions.
that. Let’s talk about it so you can handle the situation better next II. Work for meaningful learning by connecting lesson to what
time.” pupils know.
Parent F – Tells her child: “You may do what you want. We will III. Reward every child who remembers past lessons.
always be here for you, no matter what you do.” a. III only c. II and III
Which Parenting style is Authoritarian? b. I and III d. II only
a. D b. F c. E d. C
12. When small children call all animals “dogs”, what process is
3. Two identical beakers A and B are presented to the child. Teacher illustrated, based on Piaget’s cognitive development theory?
Sonny pours the liquid from B to C which is taller and thinner than a. Assimilation c. Reversion
A and B but has equal capacity with B. The teacher asks if the b. Conservation d. Accommodation
beakers A and C have the same amount of liquid. The child says
“NO” and points to C as the beaker that has more liquid. In which 13. Based on Bandura’s theory, which conditions must be present for
cognitive developmental stage is the child? a student to learn from a model?
a. Sensorimotor stage I. Attention III. Motor reproduction
b. Concrete operational stage II. Retention IV. Motivation
c. Pre-operational stage a. I and II c. I, II, III and IV
d. Formal Operational stage b. I, II and III d. III and IV

4. To determine her students’ level of moral development, Teacher


Evangeline presents to her class a morally ambiguous situation 14. According to Tolman’s theory on purposive behaviorism, learning
and asks them what they would do in such a situation. On whose is goal-directed. What is its implication to teaching?
theory is Teacher Evangeline’s technique based? a. Evaluate lessons based on your objective/s
a. Bruner b. Set as many objectives as you can
b. Kohlberg c. Stick to your objectives/s no matter what happens
c. Freud d. Make the objective/s of your lesson clear and specific
d. Piaget
15. Which is the ideal stage of moral development? Stage of _____.
5. According to R.A. 9155, which among the following is considered a. Social contract
the “heart of the formal education system”? b. Universal ethical principle
a. The pupil c. Law and order
b. The teacher d. Good boy/good girl
c. The classroom
d. The school 16. Cristina’s family had a family picture when she was not yet born.
Unable to see herself in the family picture, she cried despite her
6. You arrange the rows of blocks in such a way that a row of 5 mother’s explanation that she was not yet born when the family
blocks is longer than a row of 7 blocks. If you ask which row has picture was taken. What does Cristina’s behavior show?
more, Grade 1 pupils will say that it is the row that makes the a. Limited social cognition
longer line. Based on Piaget’s cognitive development theory, what b. Egocentrism
problem is illustrated? c. Semi-logical reasoning
a. Assimilation problem d. Rigidity of thought
b. Accommodation problem
c. Conservation problem 17. To help a student learn to the optimum, Vygotsky advises us to
d. Egocentrism problem bridge the student’s present skill level and the desired skill level by
7. According to R.A. 9155, a school head has two roles, namely ______.
administrative manager and ____. a. Challenging c. Inspiring
a. Health officer b. Scaffolding d. Motivating
b. Instructional leader
c. Facilitator 18. Based on Piaget’s theory, what should a teacher provide in the
d. Guidance counselor formal operational stage?
8. After reading and paraphrasing Robert Frost’s Stopping by the a. Stimulating environment with ample objects to play with
Woods on Snowy Evening, Teacher Marko asked the class to b. Games and other physical activities to develop motor skills
share any insight derived from the poem. In which domain in c. Activities for hypothesis formulation
Bloom’s taxonomy of objectives is the term paraphrase? d. Learning activities that involve problems of classification and
a. Analysis c. Comprehension ordering
b. Application d. Synthesis
1
19. “Do not cheat. Cheating does not pay. If you do, you cheat 28. Which among the following is closest to the real human digestive
yourself” says the voiceless voice from within you. In the context of system for study in the classroom?
Freud’s theory, which is/are at work? a. Drawing of the human digestive system on the board
a. Id c. Ego b. Model of the human digestive system
b. Id and Superego d. Superego c. The human digestive system projected on an OHP
d. Drawing of the human digestive system on a page of a
20. Here are comments from School Head Carmen regarding her textbook
observations on teacher’s practice in lesson planning:
The words “identify,” “tell” and “enumerate” are overused. Many 29. Here is a question: “Is the paragraph a good one?” Evaluate. If
times they make use of non-behavioral terms. Often their lesson broken down to simplify, which is the best simplification?
objectives do not include value formation and inculcation. a. Why is the paragraph a good one? Prove
What can be inferred from the School Head’s comments regarding b. Is the paragraph a good one? Why or Why not?
teacher formulated lesson objectives? c. If you asked to evaluate something, what do you do?
a. Often lesson objectives are in the low level Evaluate the paragraph?
b. Very often lesson objectives are in the cognitive domain d. What are the qualities of a good paragraph? Does the
c. Quite often lesson objectives describe teacher’s behavior paragraph have these qualities?
d. Often lesson objectives are in the psychomotor domain
30. Which one is in support of greater interaction?
21. Sassi, a Grade I pupil is asked, “Why do you pray everyday?” a. Probing
Sassi answered, “Mommy said so.” Based on Kohlberg’s theory, in b. Repeating the question
which moral development stage is Sassi? c. Not allowing a student to complete a response
a. Pre-convention level d. Selecting the same student respondents
b. Conventional level
c. In between conventional and post-conventional levels 31. With this specific objective, to reduce fractions to their lowest
d. In between pre- and post-conventional levels terms, this is how the teacher developed the lesson.
Step 1 – Teacher stated the rule on how to reduce fractions to
22. Teacher Fatima tells her students: “You must be honest at all their lowest term
times not only because you are afraid of the punishment but more Step 2 – Teacher wrote 2 , 3 , 4 , 5 , 6
because you yourselves are convinced of the value of honesty.” 4 6 8 10 12
Based on Kohlberg’s theory, which level of moral development and showed how to reduce them to 1
does the teacher want her students to reach? 2
a. Conventional level Step 3 – Teacher wrote 3 , 6 , 4 and
b. Between conventional and post-conventional levels 9 9 12
c. Between pre-conventional and post-conventional levels showed how to reduce them to their lowest term.
d. Post-conventional level Step 4 – Teacher gave this written exercise to the class.
Reduce the following fractions to their lowest terms: 3 , 7 , 5 , 8 ,
23. Why is babyhood referred to as a “critical period” in personality 5, 4
development? Because: 12 14 10 16 15 6
a. At this time the baby is exposed to many physical and Did the lesson begin with concrete experience then developed into
psychological hazards the abstract?
b. Changes in the personality pattern take place a. No
c. At this time the foundations are laid upon which the adult b. Yes, a little
personality structure will be built c. Yes, by way of the examples given by the teacher
d. The brain grows and develops at such an accelerated rate d. Yes, the pupils were involved in arriving at the rule on
during babyhood reducing fractions to their lowest terms
32. I want to compare two concepts. Which technique is most
24. It is good to give students creative learning tasks because ______. appropriate?
a. Development is affected by cultural changes a. Attribute wheel
b. The development of individuals is unique b. K-W-L techniques
c. Development is the individual’s choice c. Venn diagram
d. Development is aided by stimulation d. Spider web organizer
33. Which activity should a teacher have more for his students if he
25. According to Havighurst’s development tasks, reaching and wants them to develop logical-mathematical thinking?
maintaining satisfactory performance in one’s occupational career a. Focus group discussion
is supposed to have been attained during ____. b. Problem solving
a. Middle age and Early adulthood c. Games
b. Middle age d. Small group discussion
c. Old age
d. Early adulthood 34. I want to use a pre-teaching strategy that will immediately engage
my students in the content and will enable me to get an insight into
26. Student Deina says: “I have to go to school on time. This is what how students think and feel about the topic. Which is most
the rule says.” In what level of moral development is the student? appropriate?
a. Pre-conventional a. K-W-L chart c. Graphic organizer
b. Post-conventional b. Story boarding d. Document analysis
c. Conventional 35. For a discussion of a topic from various perspectives, it is best to
d. Cannot be specifically determined hold a ______.
a. Debate c. Panel discussion
27. In planning for instruction, can a teacher begin with assessment? b. Brainstorming d. Symposium
a. No, it may discourage and scare the learners 36. After establishing my learning objectives, what should I do to find
b. Yes, determine entry knowledge or skill out what my students already know and what they do not yet know
c. Yes, to make the class pay attention in relation to my lesson objectives in the cognitive domain?
d. No, assessment is only at the end of a lesson a. Give a pretest

2
b. Study the least learned competencies in the National a. Demonstration method
Achievement Test b. Type-study method
c. Analyze my students’ grades last year c. Drill method
d. Interview a sample of my students d. Laboratory method
37. What characterizes genuine change? Change in _____.
a. Appearance c. Substance 48. A teacher would use a standardized test ______.
b. Form d. Physical attribute a. To serve as a unit test
38. In which strategy, can students acquire information from various b. To serve as a final examination
perspectives, and led to reflective thinking and group consensus? c. To engage in easy scoring
a. Debate d. To compare her students to national norms
b. Small group discussion 49. Other than finding out how well the course competencies were
c. Panel discussion met, Teacher Kathy also wants to know her students’ performance
d. Symposium when compared with other students in the country. What is
39. At the end of my lesson on the role of a teacher in learning, I Teacher Kathy interested to do?
asked the class: “In what way is a teacher an enzyme?” With this a. Formative evaluation
question, it engaged the class in _______. b. Authentic evaluation
a. Allegorical thinking c. Norm-referenced evaluation
b. Concrete thinking d. Criterion-referenced evaluation
c. Metaphorical thinking 50. I want to help my students retain new information. Which one will I
d. Symbolical thinking use?
40. Which must be primarily considered in the choice of instructional a. Questions c. Games
aide? b. Mnemonics d. Simulations
a. Must stimulate and maintain student interest 51. I want to use a diagram to compare the traditional and authentic
b. Must be updated and relevant to Filipino setting modes of assessment. Which one is most fit?
c. Must be suited to the lesson objective a. Affinity diagram
d. Must be new and skillfully made b. Tree diagram
41. For lesson clarity and effective retention, which should a teacher c. Venn diagram
observe, according to Bruner’s theory? d. Fishbone diagram
a. Begin teaching at the concrete level but go beyond it by
reaching the abstract 52. A big story in your local newspaper. You want to use the headlines
b. Use purely verbal symbols in teaching as an inquiry device. To increase student participation, you might
c. Start at the concrete level and end there begin by ____.
d. End teaching with verbal symbols a. Asking one to read the news story and interpret what he read
42. Is it advisable to use realias all the time? after
a. No, for the sake of variety of instructional materials b. Asking the class to infer connotations and denotations
b. No, only when feasible from the headline
c. Yes, because there is no substitute for realias c. Explaining what you believe to be the underlying causes
d. Yes, because it is the real thing d. Describing the background of the story as you know it
43. I want my students to look at the issues on the call for President 53. If a triadic interaction includes three (3) persons, how many
Arroyo to step down from several perspectives. Which activity is persons are included in a dyadic interaction?
most fitting? a. Two
a. Cross examination c. Symposium b. Two groups composed of two persons each
b. Panel discussion d. Debate c. One, the person and himself
44. I intended to inculcate in my students the value of order and d. Four
cleanliness. I begin my lesson by asking them to share their 54. When I teach skills that are critical to the learning of the next
experiences about the dirtiest and the cleanest place they have topics, what should I employ?
seen and how they felt about them. From there I lead them to the a. Direct instruction
consequences of dirty and clean home of surroundings. In my b. Mastery learning
lesson development plan, how do I proceed? c. Socratic method
a. Transductively c. Deductively d. Cooperative learning
b. Inductively d. Concretely 55. I want my students to have mastery learning of a basic topic.
Which can help?
45. Teacher Neri wants to develop the ability of sound judgment in his a. Drill
students. Which of the following questions should he ask? b. Socratic method and drill
a. What is the essayist saying about judging other people? c. Morrisonian technique and drill
b. With the elements of a good paragraph in mind, which d. Socratic method
one is best written? 56. Teacher Feng wanted to teach the pupils the skill of cross
c. Why is there so much poverty in a country where there is stretching. Her check-up quiz was a written test on the steps of
plenty of natural resources? cross stitching. What characteristic of a good test does it lack?
d. Of the characters in the story, with whom do you identify a. Predictive validity
yourself? b. Objectivity
c. Reliability
46. The teacher is the first audio-visual aid in the classroom. What d. Content validity
does this imply? 57. In the parlance of test construction, what does TOS mean?
a. You take care that you follow the fashion or else students a. Table of Specification
won’t listen to you b. Team of Specifications
b. Your physical appearance and voice should be such that c. Table of Specifics
students are helped to learn d. Terms of Specifications
c. Make good use of the radio and TV in the classroom 58. If I favor “assessment for learning,” which will I do most likely?
d. Include singing in your teaching method I. Conduct a pre-test results
II. Teach based on pre-test results
47. I used the gumamela flower, a complete flower, to teach the parts III. Give specific feedback to students
of a flower. Which method did I use? IV. Conduct peer tutoring for students in need of help
3
a. I, II and IV c. I, II and III d. The English test appears to be the easiest among the three
b. II, III and IV d. I, II, III and IV 70. An examinee whose score is within x + 1 SD belongs to which of
59. After a lesson on the atom, the students were asked to work on a the following groups?
physical model of the atom to determine learning. For which group a. Above average
of students is building an atom model intended? b. Average
a. Interpersonality intelligent c. Below average
b. Kinesthetically intelligent d. Needs improvement
c. Mathematically intelligent 71. Which is true of a bimodal distribution?
d. Linguistically intelligent a. The scores are neither high nor low
60. If I want to develop creative thinking in my students, which one/s b. The group has two different groups
should I use? c. The score are high
I. Problem solving d. The score are low
II. Brainstorming 72. When you reach the “plateau of learning”, what should you do?
III. Dramatics a. Forget about learning
a. I and II c. III only b. Reflect what caused it
b. II only d. I, II and II c. Force yourself to learn
d. Rest
61. Which is/are effective method/s in teaching students critical 73. What can be said if student performance in a positively skewed
reading skills? scores distribution?
a. Read and interpret three different movie reviews a. Most students performed well
b. Interpret editorials about a particular subject from three b. Most students performed poorly
different newspaper c. Almost all students had averaged performance
c. Distinguish fiction from non-fiction materials d. A few students performed excellently
d. Interpret editorials and read and interpret three different 74. A number of test items in a test are said to be non-discriminating?
movie reviews What conclusion/s can be drawn?
I. Teaching or learning was very good.
62. I want to present the characteristics features of a constructivist II. The item is so easy that anyone could get it right.
approach. What should I use? III. The item is so difficult that nobody could get it.
a. Fishbone diagram a. II only c. III only
b. Venn diagram b. I and II d. II and III
c. Narrative frame 75. A test item has a difficulty index of 0.51 and a discrimination index
d. Attribute wheel of 0.25. What should the teacher do?
63. If all of your students in your class passed the pretest, what should a. Revise the item
you do? b. Retain the item
a. Administer the posttest c. Make it a bonus item
b. Go through the unit as usual because it is part of the d. Reject the item
syllabus 76. The difficulty index of a test item is 1. This means that
c. Go through the lesson quickly in order not to skip any _____________.
d. Go on to the next unit a. The test item is a quality item
64. Teacher Vicky shows her students a picture of people in thick b. The test is very difficult
jackets. Then she asks them to tell her the kind of climate when c. The test is very easy
the picture was taken. If she asks 5 questions of this kind and her d. Nobody got the item correctly
students do not get them, it is safe to conclude that pupils are 77. If the compound range is low, this means that _____________.
quite weak in the skill of _______. a. The students performed very well in the test
a. Analyzing c. Synthesizing b. The difference between the highest and the lowest score
b. Inferring d. Generalizing is low
65. Which must go with self-assessment for it to be effective? c. The difference between the highest and the lowest score is
a. Scoring rubric high
b. Consensus of evaluation results from teacher and student d. The students performed very poorly in the test
c. External monitor 78. What is the mastery level of a school division in a 100-item test
d. Public display of results of self-evaluation with a mean of 55?
66. Which group of scores is most varied? The group with ________. a. 42% b. 55% c. 45% d. 50%
a. 0.90 SD c. 0.10 SD 79. A negative discrimination index means that ____________.
b. 0.50 SD d. 0.75 SD a. The test item has low reliability
67. The main purpose in administering a pretest and a posttest to b. More from the lower group answered the test item
students is to _____. correctly
a. Measure gains in learning c. The test item could not discriminate between the lower and
b. Measure the value of the material taught upper groups
c. Keep adequate records d. More from the upper group got the item correctly
d. Accustom the students to frequent testing 80. In an entrance examination, student Anna’s Percentile is 25 (P25).
68. Assessment is said to be authentic when the teacher ________. Based on this Percentile rank, which is likely to happen?
a. Consider students’ suggestion in teaching a. Student Anna will be admitted
b. Gives valid and reliable paper-and-pencil test b. Student Anna will not be admitted
c. Gives students real-life task to accomplish c. Student Anna has 50-50 percent chance to be admitted
d. Includes parents in the determination of assessment d. Student Anna has 75 percent chance to be admitted
procedures 81. What does a percentile rank of 62 mean?
69. The following are computed means of a hundred-item test: a. It is the student’s score in the test
Physical science, 38; Math, 52; English, 33. Based on the data, b. The student answered sixty-two (62%) of the items correctly
which is true? c. The student’s score is higher than 62 percent of all
a. The examinees seem to be very good in Physical Science students who took the test
b. The Math test appears to be the easiest among the three d. Sixty-two percent (62%) of those who took the test scored
c. The examinees seem to excel in English higher than the individual
82. What does the computer have in common with the TV?
4
a. Key board c. Screen b. Jean Piaget
b. File d. Disk drive c. Howard Gardner
83. Which depicts in graphic form the social relations present in a d. Benjamin Bloom
group?
a. Interest inventory 95. Applying Confucius’s teachings, how would hiring personnel select
b. Sociogram the most fit in government positions?
c. Anecdotal record a. By record evaluation done by an accrediting body
d. Johari’s window b. By government examinations
84. Planned ignoring, signal interference, and proximity control are c. By accreditation
techniques used in _____. d. By merit system
a. Managing temper tantrums 96. “Moral example has a greater effect on pupils’ discipline than laws
b. Managing surface behavior and codes of punishment” is an advice of teachers from
c. Operant conditioning _________.
d. Life-space interviewing a. Confucius c. Lao tzu
85. What should you do to get the child’s attention when she/he is b. Mohammed d. Mencius
distracted by an object in the room? 97. “The principle of spontaneity against artificiality will make you
I. Call him by his name and tell him to pay attention accomplish something. Leave nature to itself and you will have
II. Put away the distracting influence harmony” is an active advice from the _____.
III. Involve him in helping with an activity a. Hindu c. Shintoist
a. I and II c. II and III b. Taoist d. Buddhist
b. I and III d. I, II and III 98. The significance of liberal education in holistic development of
86. Which practice in our present educational system is in line with students is supported by _____.
Plato’s thought that “nothing learned under compulsion stays with a. Perennialism
the mind”? b. Pragmatism
a. Clarification of school policies and classroom rules on Day 1 c. Confucian teaching
b. Presentation of standards of performance in the learner d. Perennialism and Confucian philosophy
c. Making the teaching-learning process interesting
d. Involving the learner in the determination of learning goals 99. Is a sick teacher, the only one assigned in a remote school,
87. In Values Education, faith, hope, and love are believed to be excused from her teaching duty?
permanent values whether they be valued by people or not. Upon a. No, she is the only one assigned in that school
what philosophy is this anchored? b. Yes, teaching is a demanding job
a. Realism c. Idealism c. Yes, she is sick
b. Existentialism d. Pragmatism d. No, she must abide by her contract
88. In the schools, we teach realities that cannot be verified by the 100. What is a demonstration of your authority as a professional
senses like an Invisible God or Allah. Whose beliefs does this teacher?
practice negate? a. You make your pupils run errands for you
a. Stoicists’ c. Skeptics’ b. You decide on how to teach a particular lesson
b. Rationalists’ d. Empiricists’ c. You absent from class to enjoy your leave even without
89. Which emphasized on non-violence as the path to true peace as prior notice
discussed in peace education? d. You select to teach only those lessons which you have
a. Taoism c. Hinduism mastered
b. Buddhism d. Shintoism 101. Which statement on true authority is wrong?
90. I make full use of the question-and-answer as a model for a. It sets an example
discussion. From whom is this question-and-answer method? b. It seeks its own satisfaction and privilege
a. Socrates c. Aristotle c. It acts in the best interest of others
b. Kant d. Plato d. Its goal is to help, form, and guide others
102. When a teacher teaches the idea that it is wrong to think that
91. A wife who loves her husband dearly becomes so jealous that in a Filipino lifestyle, products and ideas are inferior to those of other
moment of savage rage, kills him. Is the wife morally responsible nationalities, he fights against ______.
and why? a. Acculturation c. Ethnocentrism
a. Not necessarily. Antecedent passion may completely destroy b. Xenocentrism d. Culture shock
freedom and consequently moral responsibility 103. Teacher Lolita, a teacher for forty years, refuses to attend
b. Yes, she killed her husband simply because of jealousy seminars. She claims that her forty years of teaching is more
c. It depends on the case of the wife’s jealousy than all the seminars she is asked to attend. Is her actuation
d. It depends on the mental health of the wife and thinking in accordance with the Code of Ethics for
Professional Teachers?
92. Martin Luther King Jr. once said, “Man may understand all about a. No, a professional teacher, regardless of teaching
the rotation of the earth but he may still miss the radiance of the experience, ought to go through continuing
sunset.” Which type of education should be emphasized as implied professional education
by Martin Luther King Jr.? b. No, non-attendance to seminars means no professional
a. Science and education growth
b. Vocational education c. Yes, because she has taught for forty years and may
c. Liberal education have mastered the trade
d. Technical education d. Yes, provided she has an excellent performance rating

93. Computer-assisted instruction is an offshoot of the theory of 104. A principal asked her good teachers to write modular lessons in
_____. Science, and then she had them published with her name
a. J. Piaget c. B.F. Skinner printed as author. Which is unethical in this case?
b. J. Brunner d. J. Watson a. She was the exclusive beneficiary of the royalty from the
94. The use of varied teaching and testing strategies on account of modules
students’ multiple intelligences is in line with the thoughts of b. She burdened her teachers with work not related to
______. teaching
a. Daniel Goleman
5
c. She had the modular lessons published when they were c. Yes, provided they can make a bargain
worth publishing d. No, as a minority group they have the right to express their
d. She got the merit which was due for her teacher- rejection
writers 113. A teacher does not agree with the selective retention policy of
the school and she openly talks against it in her classes. Is her
105. Is it ethical on the part of the teacher to proselyte in her behavior ethical?
classroom every Friday? a. Yes, provided she got the permission from her superior to
a. Yes, that strengthens values education talk against the policy
b. Yes, that is religious instruction which is allowed by the b. No, it is her duty to faithfully carry it out even if she
Constitution does not agree
c. No, a teacher shall not engage in the promotion of c. No, in fact she is quite confused and passes on her
his/her religious interest in the classroom confusion to others
d. No, proselyting is no longer necessary in this age d. Yes, she is entitled to her opinion just as everybody is
114. If you have a problem with another teacher, the first step
106. Which can promote national pride among pupils/students? towards resolution should be for you to:
I. Studying the lives of outstanding Filipinos here and a. Talk directly with the teacher involved
abroad b. Ask your fellow teachers to intercede on your behalf
II. Reading the lives of saints of the Church c. Ask your fellow teachers for their suggestions
III. Studying Philippine history with emphasis on the victories d. Discuss it with your principal
and greatness of the Filipino people 115. What is ethical for you to do if deep within your heart you do not
a. I, II, III c. I & II agree with the school policy on student absences?
b. III only d. I only a. Be vocal about your feeling and opinion against the policy
107. Why is the exodus of Filipino professionals described as “brain b. Understand the policy and support the school in its
drain”? implementation
I. Those who go abroad are usually the better c. Argue with your superior to convince him to change the
II. Filipino professionals serve other countries instead policy
III. They contribute to nation building through their dollar d. Keep your feeling to yourself but make insinuations that
remittances you are against it while you teach
a. I, II, III b. I only c. II only d. I & II CASE #1 – Mrs. Domingo developed a lesson on the concept of
108. You want to report on a colleague's act of immorality. You don't fraction this way: First, she presented one pizza, and then asked a
have the courage to confront her. To end her illicit affair with a pupil to cut it into two. She called one part of the pizza 1/2 and the two
married man you write and secretly distribute copies of your parts of 2/2. Then she wrote 1/2 and 2/2 on the board. She proceeded to
anonymous letter against your fellow teacher. What should have ask another pupil to divide the half parts of the cake into two again, and
been done instead? then wrote 1/2, 2/4 and 3/4. Then she used the model of fractions
a. If the charge is valid; present such charge under oath (wooden circles) divided into 2, 3 and 4 show 1/2, 1/4, 1/3, 2/4. Then she
before your school head went back to the fractions she wrote on the board. She asked her
b. Ask a third party to write the anonymous letter to prevent pupils for the meaning of 1/2, 1/3, 1/4, 2/4.
yourself from being involved 116. Did Mrs. Domingo follow Bruner's three stage development of
c. Talk to the married man with whom she is having illicit knowledge?
affair a. Yes
d. Secretly give the anonymous letter only to the two people b. No
concerned c. Only in the first stage
109. Teachers often complain of numerous non-teaching d. Cannot be determined because the lesson was not
assignments that adversely affect their teaching. Does this developed fully
mean that teachers must be pre-occupied only with their 117. In the context of Bruner's principle of knowledge representation,
teaching? which is the enactive phase of the lesson on fractions?
a. Yes, because teaching is enough full time job a. Presenting the pizza and cutting it into two and four
b. Yes, if they are given other assignments, justice b. Using the model of fractions
demands that they be properly compensated c. Writing the fractions 1/2, 1/4 and 1/3 on the board
c. No, because every teacher is expected to provide d. Asking the meaning of 1/2, 1/4 and 1/3
leadership in activities for the betterment of the 118. Would it be easier to understand and retain the concept of
communities where they live and work fractions if Mrs. Domingo began the lesson on fractions with the
d. No, they are also baby sitters especially in the pre-school meaning of 1/2, 1/3, 1/4, etc.?
110. My right ends where the rights of others begin. What does this a. Yes, provided we proceed to the concrete
mean? b. No, for better learning we proceed from the concrete to
a. Rights are not absolute the abstract
b. Rights are alienable c. It depends on the teaching skills of the teacher
c. Rights are inalienable d. Yes, provided we include a concrete application of the
d. Rights are absolute abstract
111. History books used in the schools are replete with events 119. Which part of the lesson is the symbolic stage?
portraying defeats and weaknesses of the Filipino people. In the a. Using the model of fractions
spirit of nationalism, how should you tackle them in the b. Dividing the pizza further into four
classroom? c. Dividing the pizza into two
a. Present them as they are and tell the class to accept reality d. Writing the fractions 1/2, 1/4, 1/3, 2/4 on the board
b. Present the facts and use them as means to teach and 120. Does the development of the lesson on fraction conform to the
inspire your class bottom-up arrangement of the learning experiences in Edgar
c. Present the facts and express your feelings of regret Dale's Cone of Experience?
d. Present the facts including those people responsible for the a. No
failures or for those who contributed b. Cannot be determined
112. Should an association of teachers obey a Supreme Court’s c. Yes
decision even if it conflict with its interest and opinions? d. Up to the second phase of the lesson only
a. No
b. Yes
6
CASE #2 – In a faculty recollection, the teachers were asked to share c. A teacher must lord her power over her students to be an
their thoughts of the learner, their primary customer. What follow is the effective classroom manager
gist of what were shared: d. A reactive classroom management style is effective
Teacher A - “The learner is a product of his environment. Sometimes 132. Teacher Yveta involved her students in the formulation of class
he has no choice. He is determined by his environment.” rules. Which describes her classroom management style?
Teacher B - “The learner can choose what he can become despite his a. Benevolent c. Democratic
environment.” b. Authoritarian d. Laissez-faire
Teacher C - “The learner is a social being who learns well through an 133. Which adjective appropriately describes Teacher Yveta as a
active interplay with other.” classroom manager?
Teacher D - “The learner is a rational being. Schools should develop a. Proactive c. Reactive
his rational and moral powers.” b. Modern d. Traditional
121. Whose philosophical concept is that of Teacher A? 134. Was it sound classroom management practice for Teacher
a. Rationalist c. Existentialist Yveta to have involved the students in the formulation of class
b. Behaviorist d. Progressivist rules?
122. Teacher B's response comes from the mouth of a/an: a. No, it weakens the teacher's authority over her students
a. Behaviorist c. Essentialist b. Yes, it makes students feel a sense of ownership of the
b. Existentialist d. Perennialist rules
123. If you agree with Teacher C, you are more of a/an: c. Yes, it lessens a teacher's work
a. Perennialist c. Essentialist d. No, it is a students' act of usurpation of teacher’s power
b. Rationalist d. Progressivist
124. If you identify yourself with Teacher D, you adhere to what 135. Which assumption underlies Teacher Yveta's classroom
philosophy? management practice?
a. Progressivist c. Existentialist a. Classroom routines are boring and will work when imposed
b. Behaviorist d. Perennialist b. Classroom routines leave more time for class
125. Whose response denies man's free will? instruction
a. Teacher A c. Teacher D c. Students need to be treated like adults to learn
b. Teacher C d. Teacher B responsibility
CASE #3 – School Head Amilia wants her teachers to be constructivist d. Teacher's personality is a critical factor in classroom
in their teaching orientation. discipline
126. Which material will her teachers most likely use?
a. Facts c. Time-tested principles CASE #5 – Mr. Santo's lesson was on water conservation. He
b. Hypotheses d. Laws presented a graph that compared water consumption of small and big
127. Which material will her teachers most likely avoid? families. Before he asked any of the questions, he asked someone to
a. Unquestionable laws stand up to give an answer. He called only on those who raised their
b. Open-ended topics hands. The questions he asked were:
c. Problems or cases 1. What do you see in the graph?
d. Controversial issues 2. How do you compare the water consumption of small and big
128. On which assumption/s is/are the principal's action anchored? families?
I. Students learn by personally constructing meaning 3. Why do most of the big families consume more water than the
of what is taught smaller families?
II. Students construct and reconstruct meanings based on 4. Do all the small families consume less water than the big families?
experiences Explain your answer.
III. Students derive meaning from the meaning that teacher 5. In your opinion, why does one small family consume more water
gives than one big family?
a. I only c. I and II 6. In what ways is water wasted?
b. I and III d. II only 7. What are ways of conserving water?
129. Which does School Head Amilia want her teachers to do? 8. Are the families presented well at water conservation? Why or why
a. Require their students to come up with a construct of the not?
lesson 9. What generalization can you draw about water consumption and
b. Make their students derive meaning from what is presented size of families?
c. Let their students construct meaningful sentences 136. Is there any convergent question from #1 to #8?
based on the lesson a. Yes, question #4
d. Give the meaning of what they present in class b. Yes, question #7
130. Which one should her teachers then avoid? Students': c. Yes, question #8
a. Reflection d. None
b. Self-directed learning 137. Which question technique/s of Mr. Santo do(es) not enhance
c. Memorization of facts for testing interaction?
d. Inquiry I. Asking high-level questions
II. Calling only on those who raised their hands
CASE #4 – On the first day of school, Teacher Yveta oriented her class III. Calling on someone before asking the question
on procedures to be observed in passing papers, getting textbooks from a. II and III c. III only
and returning the same on the bookshelf, leaving the room for b. I and II d. I and III
necessity, and conducting group work. She arranged the chairs in such 138. Which statement on “wait time” is correct?
a way that students can interact and can move around without a. The higher the level of the question, the longer the wait
unnecessarily distracting those seated. She involved the class in the time
formulation of rules to ensure punctuality, order and cleanliness in the b. Wait time turns off slow thinking students
classroom. c. For quality response, “what” and “why” questions require
131. On what belief is Teacher Yveta's management practice equal wait time
anchored? d. Wait time discourages the brighter group of students
a. Classroom rules need to be imposed for order's sake 139. To connect the lesson on water conservation to the life of the
b. The classroom environment affects learning students, which question is most appropriate?
a. How can you help conserve water?

7
b. Based on you observations, in what ways for people 149. With the principles of learning in mind, which one can help
contribute to water wastage? Teacher Fantina solve her student disciplinary problem?
c. What maybe the reason why even if Family B is not as a. Call on their first names
big as Family C, it consumes much more water than b. Do interactive teaching
Family C? c. Change seat plan of the class
d. Among the families, who contributes most to water d. Assign monitors in class
conservation? 150. Which act of Teacher Fantina is contrary to the principles of
140. Were all the questions of Mr. Santo divergent? teaching?
a. Yes c. No a. Asking questions to check for understanding
b. No, except #4 d. Yes, except #1
b. Giving a lecture
CASE #6 – With a topic on the human circulatory system, Teacher Jan c. Checking for understanding of the lesson in the process
formulated the following lesson objectives: of teaching
1. Given a model of the human circulatory system, the student must d. Giving a test to discipline the class
be able to understand the route of blood circulation 151. Teacher Leon gives his students opportunities to be creative
2. After discussing the process of blood circulation, the teacher must because of his conviction that much learning results from the need
be able to lead the pupils in enumerating circulatory system- to express creativity. On which theory is Teacher Leon’s conviction
related diseases and in citing the causes and prevention of such anchored? ______ theory
diseases. a. Behaviorist c. Cognitive
141. Is objective #1 in accordance with the principles of lesson b. Associationist d. Humanist
objective formulation? 152. Which philosophy approves of a teacher who lectures most of the
a. No, the word “understand” is not a behavioral term time and requires his students to memorize the rules of grammar?
b. No, it is not attainable a. Existentialism c. Pragmatism
c. Yes, “understand” is an action word b. Realism d. Idealism
d. Yes, it is very specific 153. Teacher Nikka wants to check prior knowledge of her pupils about
142. How can you improve objective #2? water pollution. She writes the main topic water pollution in the
a. Remove the phrase “After discussing the process of blood center of the chalkboard and encircles it. Then, she ask the pupils
circulation.” to provide information that can be clustered around the main topic.
b. Formulate it from the learner's point of view Which technique did the teacher employ?
c. Cut it short; the statement is quite long a. Vocabulary building
d. No need to improve on it b. Semantic mapping
143. Do both objectives include a criterion of success, an element of c. Demonstration
a lesson objective cited by Robert Mager? d. Deductive teaching
a. Only objective #1 has 154. The current emphasis on the development of critical thinking by
b. Only objective #2 has the use of philosophic methods that emphasize debate and
c. No, they don't discussion began with:
d. Yes, they do a. Aristotle c. Confucius
144. Is objective #2 in accordance with the principles of lesson b. Socrates d. Plato
objective formulation? 155. Which refers to a single word or phrase that tells the computer to
a. No, it does not describe pupils' learning behavior do something with a program or file?
b. Yes, it is formulated from the point of view of the teacher a. Computer program c. Computer language
c. No, it is very broad b. Password d. Command
d. Yes, it describes teacher's teaching activity 156. In instructional planning, which among these three; unit plan,
145. Which one is the measurable learning behavior in objective #1? course plan, lesson plan is most specific? _________ plan.
a. Able to understand a. Course c. Resources
b. Route of blood circulation b. Unit d. Lesson
c. To understand 157. The first American teachers on the Philippines were:
d. None a. Missionaries
b. Soldiers
CASE #7 – Teacher Fantina has a hard time getting the attention of her c. Graduates of the normal school
class. When she checks for understanding of the lesson after a usual d. Elementary graduates
forty-five minute lecture, she finds out that only one or two can answer 158. By which process do children become participating and functioning
her questions. This has become a pattern so one time, when the class members of society by fitting into an organized way to life?
could not answer, she gave a test. a. Socialization c. Accommodation
146. What does the inattentiveness of most of Teacher Fantina's b. Acculturation d. Assimilation
students confirm? 159. What is the mean of this score distribution 4, 5, 6, 7, 8, 9, 10?
a. The “ripple effect” of behavior a. 7.5 b. 8.5 c. 6 d. 7
b. The lack of academic preparation of some teachers 160. Which is a teaching approach for kindergartens that makes real
c. The strange behaviors of today's students world experiences of the child the focal point of educational
d. The stubbornness of student groups stimulation?
a. Situation approach
147. Which method in dealing with classroom management problem b. Traditional approach
is better than that of Teacher Fantina? c. Montessori approach
a. Low level force and private communication d. Eclectic approach
b. Low level force and public communication 161. Which among the following graphic organizers used helps to show
c. High level force and private communication events in chronological order?
d. High level force and public communication a. Time line and story map
148. Can the inattentiveness of Teacher Fantina's class be attributed b. Time line and cycle
to her use of the lecture method? c. Series of events chart and story map
a. Yes, if the lecture was not interactive d. Time line and series of events chart
b. Yes, if Teacher Fantina is an experienced teacher
c. No, if the students are intelligent 162. Which schools are subject to supervision, regulation and control by
d. Cannot be determined the state?
8
a. Public, private sectarian and non-sectarian schools 174. Researchers gave rats a dose of 3-m butyl phthalide and
b. Public schools measured changes in the rats blood pressure. This statement is
c. Sectarian and non-sectarian schools best classified as
d. Private schools a. Experiment c. Hypothesis
b. Prediction d. Finding
163. Which among following can help student development the habit of
critical thinking? 175. The right hemisphere of the brain is involved with the following
a. Blind obedience of authority functions except:
b. A willingness to suspend judgment until sufficient a. Visual functions
evidence is presented b. Nonverbal functions
c. Asking convergent questions c. Intuitive functions
d. Asking low level questions d. Detail-oriented functions
176. To build a sense of pride among Filipino youth, which should be
164. Teacher Nelda wants to develop in her pupils comprehension done?
skills. What order of skills will she develop? a. Re-study our history from the perspective of our
I. Literal comprehension colonizer
II. Interpretation b. Re-study our history and stress on our
III. Critical evaluation achievements as a people
IV. Integration c. Replace the study of folklores and myths with technical
a. II-III-IV-I c. I-II-III-IV subjects
b. III-IV-I-II d. IV-III-II-I d. Set aside the study of local history
165. An integrative, conceptual approach introduced by Roldan that has
as its highest levels in the development of _____ thinking skills. 177. Which is in accordance with the “with-it-ness” principle of
a. Interpretative c. Critical classroom management of Kounin?
b. Creative d. Literal a. Students agree to disagree in class discussions
166. If a student thinks about thinking, he is involved in the process b. Teacher is fully aware of what is happening in his
called ________. classroom
a. Higher order thinking c. Student is with his teacher in everything he teaches
b. Metacognition d. Both parents and teachers are involved in the education
c. Critical thinking of children
d. Creative thinking
167. The use of drills in the classroom is rooted on Thorndike’s law of: 178. Below are questions that must be considered in developing
a. Readiness c. Exercise appropriate learning activity experiences except one. Which is it?
b. Effect d. Belongingness a. Can experiences benefit the pupils?
168. The following are some drill techniques, except: b. Do the experiences save the pupils from learning
a. Challenging students to be above the level of the class difficulties?
b. Asking pupils to repeat answers c. Are the experiences in accordance with the patterns of
c. Giving short quiz and having students grade papers pupils?
d. Assigning exercises from a workbook d. Do the experiences encourage pupils to inquire further?
169. The process of task analysis ends up in the formulation of: 179. Which is a characteristic of an imperfect type of matching test?
a. Instructional objectives a. An item may have no answer at all
b. Enabling objectives b. An answer may be repeated
c. Goals of learning c. There are two or more distracters
d. Behavioral objectives d. The items in the right and left columns are equal in
number
170. For city-bred students to think that their culture is better than those 180. Which is a proactive management practice?
from the province is a concrete example of ____. a. Tell them that you enforce the rules on everyone, no
a. Ethical relativism c. Cultural relativism exception
b. Ethnocentrism d. Xenocentrism b. Set and clarify your rules and expectation on Day 1
171. Who were the Thomasites? c. Punish the misbehaving pupils in the presence of their
a. The first American teachers that help in establishing classmates
the public educational system in the Philippines d. Stress on penalty for every violation
b. The soldiers who doubted the success of the public 181. “Specialization is knowing more and more about less and less.
educational system to be set in the Philippines Then it is better to be a generalist,” claims Teacher Patty. On
c. The first religious group who came to the Philippines on which philosophy does Teacher Patty learn?
board the US transports Thomas a. Essentialism c. Perennialism
d. The devotees to St. Thomas Aquinas who came to b. Progressivism d. Existentialism
evangelize 182. I like to develop the synthesizing skills of my students. Which one
172. Which teaching activity is founded on Bandura's social learning should I do?
theory? a. Ask my students to formulate a generalization from
a. Questioning the data shown in the graphs
b. Inductive reasoning b. Direct my students to point out which part of the graph
c. Modeling are right and which part is wrong
d. Interactive teaching c. Ask my students to answer the questions beginning with
173. Which program is DepEd’s vehicle in mobilizing support from the “what if”
private and non-government sectors to support programs based on d. Tell my students to state data presented in the graph
DepEd’s menu of assistance packages? 183. To reach out to clientele who cannot be in the classroom for one
a. Chili-Friendly-School System reason or another, which of the following was established?
b. Adopt-A-School-Program a. Informal
c. Every Child A Reader Program b. Special education (SPED)
d. Brigada Eskwela c. Pre-school education
d. Alternative learning delivery system

9
184. Tira enjoys games like scrabble, anagrams and password. Which b. Extreme personalism
type of intelligence is strong in Tira? c. “Kanya-kanya” syndrome
a. Interpersonal intelligence d. Superficial religiously
b. Linguistic intelligence 195. Teachers are encouraged to make use of authentic assessments
c. Logical and mathematical intelligence such as:
d. Spatial intelligence a. De-contextualized drills
185. Teacher Milla observes cleanliness and order in the classroom to b. Unrealistic performances
create a conducive atmosphere for learning. On which theory is c. Answering multiple choices test items
her practice based? d. Real world application of lessons learned
a. Psychoanalysis 196. What does the principle of individual difference require teachers to
b. Gestalt psychology do?
c. Behaviorism a. Give less attention to gifted learners
d. Humanistic psychology b. Provide for a variety of learning activities
186. Which learning principles is the essence of Gardner’s theory of c. Treat all learners alike while teaching
multiple intelligences? d. Prepare modules for slow learners in class
a. Almost all learners are linguistically intelligent
b. Intelligence is not measured on one form 197. The source of energy responsible for life on earth is the ______.
c. Learners have different IQ level a. Moon c. Star
d. Learners have static IQ b. Wind d. Sun
187. Who asserted that children must be given the opportunity to 198. Which quotation goes with a proactive approach to discipline?
explore and work on different materials so that they will develop a. “An ounce of prevention is better than a pound of
the sense of initiative instead of guilt? cure.”
a. Kohlberg c. Maslow b. “Do not make a mountain out of a molehill.”
b. Erickson d. Gardner c. “Walk your talk.”
188. Teacher Ada uses direct instruction strategy. Which will she first d. “Do not smile until Christmas.”
do?
a. Presenting and structuring 199. I’d like to test whether a student knows what a particular word
b. Independent practice means. What should I ask the student to do?
c. Guided student practice a. Give the word a tune then sing it
d. Reviewing the previous day’s work b. Define the word
189. The free public elementary and secondary educations in the c. Spell the word and identify its part of speech
country are in the line with the government effort to address d. Give the etymology of the word
educational problems of _________.
a. Productivity 200. Study this matching type of test. Then answer the question below.
b. Relevance and quality 1. Measure of relationship a. Mean
c. Access and quality 2. Measure of central b. Standard Deviation
d. Effectiveness and efficiency tendency c. Rho
190. Here are raw scores in a quiz 97, 95, 85, 83, 77, 75, 50, 10, 5, 2, 3. Binet-Simon d. T-ratio
1. To get a picture of the group’s performance which measure of 4. Statistical test of mean e. Intelligence testing
central tendency is most reliable? difference movement
a. Median 5. Measure of variability
b. Mode Which among the following is a way to improve the above
c. Mean matching test?
d. None, it is best to look at the individual scores a. Add five items in both columns
b. Add one or two items in the right column
191. Which objective in the affective domain is the lowest level? c. Add ten items in both columns to make the test more
a. To accumulate examples of authenticity comprehensive
b. To support viewpoints against abortion d. Add one or two items in the left column
c. To respond positively to a comment
d. To formulate criteria for honestly 201. A teacher’s quarrel with a parent makes her develop a feeling of
192. “Using the six descriptions of elements of good short story, prejudice against the parent’s child. The teacher’s unfavorable
IDENTIFY IN WRITING THE SHORT STORY BY O. HENRY, with treatment of the child is an influence of what Filipino trait?
complete accuracy.” The words in capital letters are referred to as a. Lack of self-reflection
the ____________. b. Extreme personalism
a. Criterion of success c. Extreme family-centeredness
b. Condition d. “Sakop-mentality”
c. Performance statement
d. Minimum acceptable performance 202. In the context of grading, what is referred to as teacher’s
generosity error? A teacher _______.
193. You have presented a lesson on animal protective coloration. At a. Rewards students who perform well
the end, you ask if there are any questions. There are none. You b. In overgenerous with praise
can take this to mean that ______. c. Has a tendency to give high grades as compare to
a. The students are not interested in the lesson the rest
b. You need to ask specific questions to elicit d. Gives way to students’ bargain for no more quiz
responses 203. For mastery learning and in line with outcome-based evaluation
c. The students did not understand what you were talking model which element should be present?
about a. Inclusion of non-performance objectives
d. The students understood everything you presented b. Construction of criterion-referenced tests
194. Teacher Vina feels offended by her supervisor’s unfavorable c. Construction of norm-referenced tests
comments after a classroom supervision. She concludes that her d. Non-provision of independent learning
supervisor does not like her. Which Filipino trait is demonstrated 204. Which statement about standard deviation is correct?
by Teacher Vina?
a. Extreme family-centeredness
10
a. The higher the standard deviation the more spread d. Prepare students for law-making
the scores are 216. Which goals of educational institution, as provided for by the
b. The lower the standard deviation the more spread the Constitution, is the development of work skills aligned?
scores are a. To develop moral character
c. The higher the standard deviation the less spread the b. To develop vocational efficiency
scores are c. To teach the duties of citizenship
d. It is a measure of central tendency d. To inculcate love of country
205. What is the Teacher’s Professionalism Act? 217. Complete this analogy:
a. RA 7836 c. RA 7722 Spanish period: moral and religious person.
b. RA 4670 d. RA 9263 American period: _______
206. The State shall protect and promote the right of citizens to quality a. Productive citizen c. Patriotic citizen
education at all levels. Which government program is in support of b. Self-reliant citizen d. Caring citizen
this? 218. Who is remembered for his famous quotation? “My loyalty to may
a. Exclusion of children with special needs from the formal party ends where my loyalty to my country begins.”
system a. Carlos P. Garcia c. Manuel L. Quezon
b. Free elementary and secondary education b. Ferdinand Marcos d. Manuel Roxas
c. Deregulated tuition fee hike 219. Here is a question: “Is the paragraph a good one? Evaluate.” If
d. Re-introduction of the NEAT and NSAT broken down to simplify, which is the best simplification?
207. The American Teachers who were recruited to help set the public a. Is the paragraph a good one? Why or why not?
educational system in the Philippines during the American regime b. Why is the paragraph a good one? Prove
were called Thomasite’s because: c. If you asked to evaluate something, what do you do?
a. They were devotees of St. Thomas Aquinas Evaluate the paragraph
b. They disembarked from the CIS Transport called d. What are the qualities of a good paragraph? Does the
Thomas paragraph have these qualities?
c. They first taught at the University of Sto. Tomas 220. Manunulat ang tatay mo kaya sa personal mong kagustuhan
d. They arrived in the Philippines on the feast of St. Tomas makasunod sa kanyang mga yapak, magsusulat ka. Ano ang
208. Zazha exhibits fear response to freely roaming dogs but does not kahulugan ng pagsusulat?
show fear when a dog is on a leash or confined to a pen. Which a. Proseso na nagsisimula sa idea o karanasan
conditioning process is illustrated? b. Makalikha ng mga salita, pangungusap at talata
a. Generalization c. Discrimination c. Kakayahan ng tao na makapagtala o makapagimprenta
b. Acquisition d. Extinction ng malinaw
209. The Filipino learner envisioned by the Department of Education is d. Pagpili ng paksa at pagsasaliksik
one who is imbued with the desirable values of person who is: 221. Anong bahagi ng pananalita ang nasa malaking titik? Malungkot
a. Makabayan, makatao, makakalikasan at maka-Diyos ANG MGA nagtapos na wala pang trabaho.
b. Makabayan, makatao, makahalaman, at maka-Diyos a. Pananda c. Pariralang pantukoy
c. Makabayan, makasarili, makakalikasan, at maka-Diyos b. Pang-ukol d. Pangatnig
d. Makabayan, makakaragatan, makatao, at maka-Diyos 222. Which is NOT a characteristic of education during the pre-Spanish
210. The right hemisphere of the brain is involved with the following era?
function except: a. Vocational training-oriented
a. Intuitive functions b. Structured
b. Nonverbal functions c. Unstructured
c. Visual functions d. Informal
d. Detail-oriented functions 223. Which is closest to the real human digestive system for study in
211. What function is associated with the right brain? the classroom?
a. Visual, non-verbal, logical a. Model of the human digestive system
b. Visual, intuitive, non-verbal b. Drawing of the human digestive system on the board
c. Visual, intuitive, logical c. The human digestive system projected on an OHP
d. Visual, logical, detail-oriented d. Drawing of human digestive system on a page of a
212. Principal Connie tells her teachers that training in the humanities is textbook
most important. To which education philosophy does he adhere? 224. Which one is in support of greater interaction?
a. Existentialism c. Essentialism a. Repeating the question
b. Progressivism d. Perennialism b. Not allowing a student to complete a response
213. Behavior followed by pleasant consequences will be strengthened c. Probing
and will be more likely to occur in the future. Behavior followed by d. Selecting the same student respondents
unpleasant consequences will be weakened and will be less likely 225. Which is/are effective methods/s in teaching students critical
to be repeated in the future. Which one is explained? reading skills?
a. Freud’s psychoanalytic theory a. Interpret editorials about a particular subject from three
b. Thorndike’s law of effect different newspapers
c. B.F. Skinner’s Operant conditioning theory b. Read and interpret three different movie reviews
d. Bandura’s social learning theory c. Distinguish fiction from non-fiction materials
214. Theft of school equipment like TV, computer, etc. by people on the d. Interpret editorials and read and interpret three
community itself is becoming a common phenomenon. What does different movie reviews
this signify? 226. Which is true of a bimodal score distribution?
a. Prevalence of poverty in the community a. The group tested has two different groups
b. Inability of school to hire security guards b. The scores are neither high nor low
c. Deprivation of Filipino schools c. The scores are high
d. Community’s lack of sense of co-ownership d. The scores are low
215. The main purpose of compulsory study of the Constitution is to 227. What is the mastery level of school in a 100 item test with a mean
_____. of 55?
a. Develop students into responsible, thinking citizen a. 42% b. 50% c. 45% d. 55%
b. Acquaint students with the historical development of the
Philippine Constitution
c. Make constitutional experts of the students
11
228. Who stressed the idea that students cannot learn if their basic c. The law of inertia
needs are not first met? d. Rules on subject-verb agreement
a. Thorndike 240. Teacher Joshua discovered that his pupils are weak in
b. Maslow comprehension. To further in which particular skill(s) his pupils are
c. Wertheimer weak, which test should Teacher Joshua give?
d. Operant conditioning a. Standardized test c. Placement
b. Aptitude test d. Diagnostic
229. A person, who has had painful experience as the dentist’s office, 241. The students of Teacher Kath scan an electronic encyclopedia,
may become fearful at the mere sight of the dentist’s office view a film on the subject, or look at related topics at the touch of a
building. What theory can explain this? button right there in the classroom. Which devices does teacher
a. Attribution theory Kath’s class have?
b. Classical conditioning a. Videotape lesson c. Video disc
c. Generalization b. Teaching machine d. CD
d. Operant conditioning 242. If my approach to my lesson is behaviorist, what features will
dominate my lesson?
230. I want my students to have mastery learning of a basic topic. I. Lecturing III. Reasoning
Which of the following can help? II. Copying notes IV. Demonstration
a. Socratic method and drill a. III, IV c. I, II, III, IV
b. Drill b. I, III, IV d. II, III, IV
c. Socratic method 243. You practice inclusive education. Which of these applies to you?
d. None of the above I. You accept every student as full and valued member of the
231. As a teacher you are a skeptic. Which among these will be your class and school community
guiding principle? II. Your special attention is on learners with specific learning or
a. I must teach the child to the fullest social needs
b. I must teach the child every knowledge, skill, and value that III. You address the needs of the class as a whole within the
he needs for a better future context of the learners with specific learning or social needs
c. I must teach the child that we can never have real a. II only c. I only
knowledge of anything b. I and II d. I and III
d. I must teach the child so he is assured 244. Which will be the most authentic assessment tool for an
232. With forms of prompting in mind, which group is arranged from instructional objective on working with and relating to people?
least to most instructive prompting? a. Organizing a community project
a. Verbal, physical, gestural b. Writing articles on working and relating to people
b. Verbal, gestural, physical c. Home visitation
c. Gestural, physical, verbal d. Conducting mock election
d. Physical, gestural, verbal 245. Here is a test item:
233. In which way does heredity affect the development of the learner? “From the data presented in the table, form generalizations that
a. By providing equal potential to all are supported by the data.” Under what type of question does this
b. By making acquired traits hereditary item fall?
c. By compensating for what environment fails to develop a. Convergent c. Application
d. By placing limits beyond which the learner cannot b. Evaluative d. Divergent
develop 246. I want to teach concepts, patterns, and abstractions. Which
234. In writing performance objectives, which word is not acceptable? method will be most appropriate?
a. Integrate c. Manipulate a. Discovery c. Direct instruction
b. Delineate d. Comprehend b. Indirect instruction d. Problem solving
235. For which lesson objective will a teacher use the direct instruction 247. We are very much interested in a quality professional development
method? program for teachers. What characteristic should we look for?
a. Distinguish war from aggression a. Required for renewal of professional license
b. Appreciate Milton’s Paradise Lost b. Prescribed by top educational leader
c. Use a microscope properly c. Responsive to identified teacher’s needs
d. Become aware of the pollutants in the environment d. Dependent on the availability of funds
236. To encourage introspection, which teaching method is MOST 248. What principle is violated by overusing the chalkboards, as though
appropriate? it is the only education technology available?
a. Cognitive c. Process a. Isolated use c. Variety
b. Reflective d. Cooperative learning b. Flexibility d. Uniformity
237. With indirect instruction in mind, which does not belong to the 249. Which statement applies correctly to Edgar Dale’s “CONE of
group? experience”?
a. Lecture-recitation a. The farther you are from the base, the more direct the
b. Experiential method learning experience becomes
c. Inductive method b. The farther you are from the bottom, the more direct the
d. Discovery method learning experience becomes
238. A master teacher is the resource speaker in an in-service training. c. The close you are to the base, the more indirect the
He presented the situated learning theory and encouraged his learning experience become
colleagues to apply the same in class. Which of the following did d. The closer you are to the base, the more direct the
he not encourage his colleagues to do? learning experience becomes
a. Apprenticeship 250. To teach the democratic process to the pupils. Santo Domingo
b. Decontextualized teaching Elementary School decided that the election of class officers
c. Learning as it normally occurs shall be patterned after local elections. There are qualities set
d. Authentic problem solving for candidates, limited period for campaign, rules for posting
239. I want to engage my students in small group discussions. Which campaign materials, etc. Which of the following did the school
topic lends itself to a lively discussion? use?
a. The exclusion of Pluto as a planet a. Symposium c. Role playing
b. The meaning of the law of supply and demand b. Simulation d. Philips 66

12
251. Which of the following are effective methods in teaching student 260. Teacher Nene asked this question: “What conclusion can you
critical reading skills? draw based on your observation?” Nobody raised a hand so she
I. Interpret editorials asked another question: “Based on what you observe, what can
II. Read and interpret three different movie reviews you now say about the reaction of plants to light.” What did
III. Read a position paper and deduce underlying assumptions Teacher Nene do?
of the position papers a. Redirecting c. Repeating
a. II and III c. I and II b. Probing d. Rephrasing
b. I and III d. I, II and III
252. Here is a test item: 261. Under no circumstance shall a teacher be prejudiced nor
“The improvement of basic education should be the top priority discriminatory against any learner according to the Code of
of the Philippine government. Defend and refute the position.” Ethics. When is a teacher prejudice against any learner?
Under what type of question does this test item fall? a. When he makes a nearsighted pupil sit at the front
a. Low-level c. Analysis b. When he considers multiple intelligences in the choice of
b. Evaluative d. Convergent his teaching strategies
253. Teacher Jenny teaches a lesson in which students must c. When he makes a farsighted pupil sit at the back
recognize that ¼ is the same as 0.25. They use this relationship d. When he refuses a pupil with a slight physical
to determine that 0.15 and 0.20 are slightly less than ¼. Which disability in class
of the following concept/s is/are being taught?
a. Numeration skills 262. Which learning activity is most appropriate if teacher’s focus in
b. Place value of decimals attitudinal change?
c. Numeration skills for decimal and relationship between a. Role play c. Exhibit
fractions and decimals b. Field trip d. Game
d. Relationship between fractions and decimals 263. The mode of a score distribution is 25. This means that:
254. To nurture student’s creativity, which activity should the teacher a. There is no score of 25
avoid? b. Twenty five (25) is the score that occurs most
a. Ask “what if” questions c. Twenty five is the average of the score distribution
b. Ask divergent thinking questions d. Twenty five is the score that occurs least
c. Emphasize the need to give right answer 264. The following characterize a child-centered kindergarten except:
d. Be open to “out-of-this world” ideas a. Focus on the education of the whole child
b. Importance of play in development
255. After reading an essay, Teacher Bebe wants to help sharpen c. Extreme orientation on academic
her students’ ability to interpret. Which of these activities will be d. Emphasis on individual uniqueness
most appropriate? 265. As a classroom manager, how can you exhibit expert power on
a. Drawing conclusions the first day of school?
b. Making inferences a. By citing to my students the important of good grades
c. Getting the main idea b. By making my students feel my authority over them
d. Listing facts separately from opinion c. By making them feel a sense of belongingness and
acceptance
256. What is the best way to develop math concepts? d. By making my students feel I know what I am talking
a. Solving problems using multiple approaches about
b. Solving problems by looking for correct answer 266. Which may help an adolescent discover his identity?
c. Learning math as applied to situations such as being a a. Parents pushing in to follow a specific path
tool of science b. Relating to people
d. Solving problems by applying learned formulas c. Decision to follow one path only
257. Teacher Cita, an experienced teacher, does daily review of past d. Exploring many different roles in a healthy manner
lessons in order to ________. 267. Which terms refers to a teacher helping a colleague grow
a. Provide her pupils with a sense of continuity professionally?
b. Introduce a new lesson a. Technology transfer
c. Reflect on how she presented the previous lesson b. Peer mentoring
d. Determine who among her pupils are studying c. Facilitating
d. Independent study
258. Research says that mastery experiences increase confidence 268. What primary criterion should guide a teacher in the choice of
and willingness to try similar or more challenging tasks such as instructional devices?
reading. What does this imply for children reading performance? a. Novelty c. Appropriateness
a. Children who have mastered basic skills are more likely to b. Cost d. Attractiveness
be less motivated to read because they get fed up with too 269. Some of your students don’t seem to like you as their teacher. If
much reading you will regard the situation, on the level of the ego, what will
b. Children who have not mastered the basic skills are more you most likely think about?
likely to be motivated to read in order to gain mastery over a. Why should I care if they like me or not
basic skills b. To hell with them
c. Children who have a high sense of self-confidence are not c. What’s wrong have I done to deserve this?
necessarily those who can read d. What is it about me that they do not like?
d. Children who have gained mastery over basic skills are 270. Which of the following is considered a peripheral device?
more motivate to read a. Printer c. CPU
b. Keyboard d. Monitor
259. Bruner’s theory on intellectual development moves from 271. Which questioning practice will promote more class interaction?
enactive to iconic and symbolic stages. Applying Bruner’s a. Asking rhetorical question
theory, how would you teach? b. Rejecting wrong answer
a. Begin with the abstract c. Focusing on convergent question
b. Be interactive in approach d. Asking divergent question
c. Begin with the concrete 272. For grades to be (made) valid indicators of students’
d. Do direct instruction achievements, which process should be observed?
a. Adopting letter grades such as A, B, C, D
13
b. Explaining the meaning of grades d. Appreciation of manual labor
c. Defining the course objectives as intended learning 284. Which of the following does extreme authoritarianism in the
outcomes home reinforce in pupils?
d. Giving objective type of test a. Sense of initiative
273. The claim of a benefactor to the gratitude of his protégé is an b. Dependence on other for direction
example of a (an): c. Ability for self-direction
a. Acquired right c. Imperfect right d. Creativity in work
b. Perfect right d. Alienable right 285. Teacher Bart wants his students to master the concept of social
274. Which appropriate teaching practice flows from this research justice. Which series of activities will be most effective?
finding on the brain: “The brain’s emotional center is tied its a. Pre-teaching > posttest > re-teaching of unlearned concept
ability to learn.” > posttest
a. Create a learning environment that encourages b. Pre-test > teaching > posttest
students to explore their feelings and ideas freely c. Review > pretest > teaching > posttest
b. Come up with highly competitive games where winners will d. Teaching > posttest
feel happily 286. To provide for individual differences how is curriculum
c. Establish this discipline of being judgmental in attitude designed?
d. Tell the students to participate in class activities or else a. Minimum learning competencies are included
they won’t receive plus points in class recitation b. Realistic and meaningful experiences are provided
275. Research on Piagetian tasks indicates that thinking becomes c. Some degree of flexibility is provided
more logical and abstract as children reach the formal d. Social skills are emphasized
operations stage. What is an educational implication of this 287. Which types of play is most characteristic of a four- to six-year-
finding? old child?
a. Expect hypothetical reasoning for learners between 12 a. Solitary and onlooker plays
to 15 years of age b. Associative and cooperative plays
b. Learners who are not capable of logical reasoning from c. Associative and onlookers plays
ages 8 to 11 behind in their cognitive development d. Cooperative and solitary plays
c. Engage children in analogical reasoning as early as 288. How can you exhibit referent power on the first day of school?
preschool to train them for higher order thinking skills a. By making the students feel you know what you are talking
(HOTS) about
d. Let children be children b. By telling them the importance of good grades
276. A mathematics test was given to all Grade V pupils to determine c. By reminding your students your authority over them
the contestants for the math quiz bee. Which statistical measure again and again
should be used to identify the top 15? d. By giving your students a sense of belonging and
a. Percentage score acceptance
b. Mean percentage score 289. To ensure that all Filipino children are functionally literate, which
c. Quartile score mechanism is meant to reach out to children who are far from a
d. Percentile score school?
277. Which is the true foundation of the social order? a. A school in every barangay
a. Strong, political leadership b. Multi-grade classroom
b. The reciprocation of rights and duties c. Mobile teacher
c. Equitable distribution of wealth d. Sine’s skwela
d. Obedient citizenry 290. Referring to the characteristics of the latest Basic Educational
278. With which is true authority equated? Curriculum which does not belong to the group?
a. Service c. Suppression a. More flexible
b. Power d. Coercion b. Less prescriptive
279. The following are sound specific purposes of questions except: c. More compartmentalized
a. To stimulate learners to ask questions d. More integrated
b. To call the attention of an inattentive student 291. If student’s inappropriate behavior is low level or mild and that it
c. To arouse interest and curiosity appears that the misbehavior will not spread to others, it is
d. To teach via student answers sometimes best for the teacher not to take notice of it. What
280. What is not a sound purpose in making questions? influence technique is this?
a. To remind students of a procedure a. Planned ignoring
b. To probe deeper after an answer is given b. Antiseptic bouncing
c. To encourage self-reflection c. Proximity control
d. To discipline a bully in class d. Signal interference
281. A student passes a book report written but ornately presented in 292. Which among the following objectives in the psychomotor
a folder to make up for the poor quality of the book content. domain is highest in level?
Which Filipino trait does this practice prove? a. To distinguish distant and close sounds
a. Art over science b. To contract a muscle
b. Art over academic c. To run a 100-meter dash
c. Substance over “porma” d. To dance the basic steps of the waltz
d. “Porma” over substance 293. Which material consists of instructional units that cater to
282. Which one should a teacher avoid to produce an environment varying mental level pupils?
conducive for learning? a. Plantilia
a. Games c. Tests b. Multi-level materials
b. Seat plan d. Individual competition c. Multi-grade materials
283. Between pursuing a college course where there is no demand d. Minimum learning competencies
and a vocational course which is highly in demand, the Filipino 294. Which statement on counseling is false?
usually opts for the college course. Which Filipino trait is a. For counseling to be successful, the counselee is willing to
shown? participate in the process
a. Interest to obtain a skill b. The ultimate goal of counseling is greater happiness
b. Penchant for a college diploma on the part of the counselee
c. Desire for entrepreneurship c. Counseling is the program that includes guidance
14
d. The school counselor is primarily responsible of counseling I. A school head plans the professional development of his/her
295. Arianna describes Teacher Monica as “fair, caring and teachers.
approachable.” Which power does Teacher Monica possess? II. Every teacher formulates his/her own professional
a. Legitimate power development plan.
b. Expert power III. The implementation of what is learned in training must be
c. Referent power monitored.
d. Reward power a. I only c. II and III
296. Kounin claims “with-it-ness” is one of the characteristics of an b. I and III d. II only
effective classroom manager. Which among the following is a 305. As a community leader, which of the following should a teacher not
sign of “with-it-ness”? do?
a. Giving attention to students having difficulty with school a. Solicit donation from philanthropists in the community
work b. Support effort of the community to improve their status in life
b. Seeing only a portion of the class but intensively c. Make herself aloof to ensure that her decisions will not
c. Knowing where instructional materials are kept be influenced by the community politics
d. Aware of what’s happening in all part of the classroom d. Play an active part in the activities of the community
297. In the K-W-L technique, K stands for what the pupils already 306. In a highly pluralistic society, what type of learning environment is
knows, W for what he wants to know and L for what he: the responsibility of the teacher?
a. Learned c. Failed to learn I. Safe III. Secure
b. He like to learn d. Needs to learn II. Gender-biased
298. When a significantly greater number from the lower group gets a a. I and II c. II only
test item correctly, this implies that the test item: b. I, II and III d. I and III
a. Is not highly reliable 307. A teacher is said to be “trustee of the cultural and educational
b. Is not very valid heritage of the nation and is under obligation to transmit to
c. Is highly reliable learners such heritage”. Which practice makes the teacher fulfill
d. Is very valid such obligation?
299. Which statement about guidance is false? a. Use interactive teaching strategies
a. The classroom teacher is not part of the school
b. As a class, study the life of Filipino heroes
guidance program since she is not trained to be a
c. Use the latest educational technology
guidance counselor
b. Guidance embraces curriculum, teaching, supervision and d. Observe continuing professional education
all other activities in school
c. Guidance is a function of the entire school 308. Large class size in congested cities is a common problem in our
d. A guidance program is inherent in every school public schools. Which measure/s have schools taken to offset the
300. A teacher should not be a slave of his lesson plan. This means effects of large class?
that: I. The deployment of more teachers
a. A teacher must be willing to depart from her lesson II. The implementation of 1:1 pupil textbook ratio
plan if students are interested in something other than III. The conduct of morning and afternoon sessions
her intended lesson a. I, II and III c. III only
b. A lesson plan must be followed by a teacher no matter b. I and II d. II only
what 309. The failure of independent study with most Filipino students may
c. A teacher must be ready to depart from her lesson plan if be attributed to students’?
she remembers something more interesting than what she a. Unpreparedness for schooling
earlier planned b. Ambivalence
d. Teacher is the best lesson plan designer c. High degree of independence
301. With a death threat over her head, Teacher Donita is directed to d. High degree of independence on authority
pass an undeserving student. What will a utilitarianist do?
a. Pass the student, why suffer the threat? 310. The following are characteristics of interdisciplinary teaching
b. Pass the student. That will be of use to the student, his except:
parents and you a. Allows learners to see connectedness between things
c. Don’t pass him; live by your principle of justice. You will get b. Provides learning opportunities in a real-world context
reward, if not in this life, in the next c. Eliminates boundaries between content area
d. Don’t pass him. You surely will not like someone to give you a d. Discussion from a single perspective
death threat in order to pass 311. For more efficient and effective management of school as agents
302. Teacher Alessandra knows of the illegal activities of a neighbor but of change, one proposal is for the DepEd to cluster remote stand-
keeps quiet in order not to be involved in any investigation. Which alone schools under one lead school head. Which factor has the
foundational principle of morality does Teacher Alessandra fail to strongest influence on this proposal?
apply? a. Psychological c. Geographical
a. Always do what is right b. Historical d. Social
b. The end does not justify the means 312. What does the acronym EFA imply for schools?
c. The end justifies the means a. The acceptance of exclusive schools for boys and for girls
d. Between two evils, do the lesser evil b. The stress on the superiority of formal education over that of
303. You are very much interested in a quality professional alternative learning system
development program for teachers. What characteristic should you c. Practice of inclusive education
look for? d. The concentration on formal education system
a. Prescribe by top educational teachers
b. Responsive to identified teacher’s needs 313. The wide acceptance of “bottom up” management style has
c. Dependent on the availability of funds influenced schools to practice which management practice?
d. Required for renewal of professional license a. Exclusion of politicians from the pool of guest speakers during
304. To ensure high standards of teachers’ personal and professional graduation exercises
development, which of the following measures must be b. Prescription of what ought to be done from the Center Office
implemented? c. Involvement of students, parents, teachers, and
community in school planning
15
d. Allowing schools to do what they think is best c. Yes, it supports equitable access to basic education
314. Material development at the expense of human development d. No, it does not support parent of adult education
points to the need to do more in school. This is base on which
pillar of learning? 323. “Specialization is knowing more and more about less and less.”
a. Learning to do Hence, it is better to be a generalist, claims Teacher Fonda. Which
b. Learning to know Philosophy does Teacher Fonda subscribe to?
c. Learning to live together a. Existentialism c. Essentialism
d. Learning to be b. Perennialism d. Progressivism

315. The schooling incidents in school campuses abroad have made 324. Mencius believed that all people are born good. This thought on
school to rethink the curriculum. Which is believed to counteract the innate goodness of people makes it easier to ________ our
such incidents and so is being introduced in schools? pupils.
I. Inclusion of socio-emotional teaching a. teach c. like
II. The emphasis on the concept of competition against self and b. respect d. motivate
not against others 325. A student complains to you about his failing grade. When you
III. Focus on academic achievement and productivity recomputed you found out that you committed an error in his grade
a. I and III c. I and II computation. Your decision is not to accept the erroneous
b. II and III d. I, II and III computation before the student and so leave the failing grade as is
for fear that you may lose credibility. Is this morally right?
316. The specialization requires of every professional teacher for a. No, the reason for not accepting the error before the students
him/her to be competent is in line with which pillar of learning? is flimsy
a. Learning to know b. No, the end does not justify the means
b. Learning to be c. Yes, the end justifies the means
c. Learning to live together d. Yes, as a teacher you must maintain your credibility
d. Learning to do 326. Which violate(s) the principle of respect?
317. Widespread abuse of Mother Earth prompted schools to teach I. Teacher Ana tells her students that what Teacher Beth taught
sustainable development. Which one does this prove about is wrong.
schools? II. To retaliate, Teacher Beth advises students not to enroll in
a. The curricula of schools are centered on Mother Earth Teacher Ana’s class.
b. Environment factors influence the school as an agent of III. Teacher Catherine secretly gives way to a special favor (e.g.
change add 2 points to grade) requested by student Alex who is vying
c. Schools can easily integrate sustained development in their for honors.
curriculum a. II and III c. I and II
d. Sustained development cannot be effectively taught in the b. I, II and III d. I and III
classroom 327. Which is/are in accordance with the principle of pedagogical
318. Which pillar of learning is aimed at the holistic development of man competence?
and his complete fulfillment? I. Communication of objectives of the course to student.
a. Learning to live together II. Awareness of alternative instruction strategies.
b. Learning to be III. Selection of appropriate methods of instruction.
c. Learning to know a. I and III c. III only
d. Learning to do b. I, II and III d. II and III
319. A father tells his daughter “You are a woman. You are meant for 328. To earn units for promotion, Teacher Flora pays her fee but does
the home and so for you, going to school is not necessary.” Is the not attend class at all. Does this constitute professional growth?
father correct? a. Not immediately but yes after promotion
a. It depends on the place where the daughter and the father b. No, it is simply earning MA units for promotion
live c. It depends on the school she is enrolled in
b. No, there is gender equality in education d. Yes, just enrolling in an MA program is already professional
c. Yes, women are meant to be a mother only growth
d. No, today women can take on the jobs of men 329. If Teacher Analiza asks more higher-order questions, she has to
ask more ________ questions.
320. Is there a legal basis for increasing the teacher’s starting salary to a. fact c. convergent
PHP18,000 a months? b. close d. concept
a. No, it is a gift to teachers from Congress 330. Misdemeanor has a “ripple effect.” This implies that as a
b. Yes, R.A 7836 classroom manager, a teacher:
c. Yes, the Philippine Constitution a. reinforces positive behavior
d. No, it is simply an act of benevolence from President GMA b. responds to misbehavior promptly
321. As provided for the Educational Act of 1982, how are the c. is consistent in her classroom management practice
institutions of learning encouraged setting higher standards of d. count 1 to 10 before she deals with a misbehaving student
equality over the minimum standards required for state 331. Based on Edgar Dale’s “Cone of Experience,” which activity is
recognition? farthest from the real thing?
a. Voluntary accreditation a. Watching demo c. Video disc
b. Continuing Professional Education b. Attending exhibit d. Viewing images
c. Granting of Special Permit 332. The students of Teacher Yue scan an electronic encyclopedia,
d. Academic freedom view a film on subject, or look at related topics at the touch of a
button right there in the classroom. Which device/s does teacher
322. Despite of opposition from some school official, DepEd has Yue’s class have?
continuously enforced the “no collection of fees” policy during a. Teaching machines
enrolment period in public schools. In this policy in accordance b. CD
with EFA goals? c. Video disc
a. No, it violates the mandate of equality education d. Videotaped lesson
b. Yes, it somewhat eliminates gender disparities 333. Which is an inappropriate way to manage off-task behavior?
16
a. Redirect a child’s attention to task and check his progress to d. reminding your students your authority over them again and
make sure he is continuing work again
b. Stop your class activity to correct a child who is no 345. I would like to use a model to emphasize particular part. Which of
longer on task these would be most appropriate?
c. Make eye contact to communicate what you wish to a. Regalia c. Stimulation
communicate b. Audio recording d. Mock up
d. Move closer to the child to make him feel your presence
334. When Teacher Pearl tries to elicit clarification on a student 346. What must Teacher Luke do to ensure orderly transitions between
response or solicits additional information, which of these should activities?
be use? a. Have the materials ready at the start of the activity
a. Directing c. Structuring b. Allow time for the students to socialize in between activities
b. Probing d. Cross examining c. Assign fewer exercise to fill the allotted time
335. Which priority criterion should guide a teacher in the choice of d. Wait for students who lag behind
instructional devices? 347. The task of setting up routine activities for effective classroom
a. Novelty c. Attractiveness management is a task that a teacher should undertake:
b. Cost d. Appropriateness
336. Which learning activity is most appropriate if a teacher’s focus is a. as soon as the students have adjusted to their schedules
attitudinal change? b. on the very first day of school
a. Fieldtrip c. Role play c. every day at the start of the session
b. Exhibit d. Game d. every homeroom day
337. Teacher Hannah strives to draw participation of every student into
her classroom discussion. Which of these student needs is she 348. What principle is violated by overusing the chalkboard, as though it
trying to address? The need to _______. is the only education technology available?
a. feel significant and be part of a group a. Isolated use c. Variety
b. show one’s oral abilities to the rest of the class b. Flexibility d. Uniformity
c. get everything and be part of a group
d. be creative 349. Teacher Sandra uses the low-profile classroom control technique
338. To be an effective classroom manager, a teacher must be friendly most of the time. What does this imply?
but must at the same time be _________. a. She is reactive in her disciplinary orientation
a. confident c. analytical b. She manages pupils personalities
b. business-like d. buddy-buddy c. She reacts severely to a misbehaving student
d. She stops misbehaving without disrupting lesson flow
339. Which software is needed when one wants to perform automatic
calculations on numerical data? 350. If curriculum is designed following the traditional approach, which
a. Database feature(s) apply(ies)?
b. Spreadsheet Program I. The aims of the curriculum are set by professionals and
c. Microsoft Word experts.
d. Microsoft Powerpoint II. Interested groups (teachers, students, communities) are
assumed to agree with the aims of the curriculum.
340. Which of the following questions must be considered in evaluating III. Consensus building in not necessary.
teacher-made materials? a. III only c. I and II
a. In the material new? b. I, II and III d. I and III
b. Does the material simulate individualism?
c. Is the material expensive? 351. If you make use of the indirect instruction method, you begin your
d. Is the material cheap? lesson with:
341. Kounin claims that “with-it-ness” is one of the characteristics of an a. Guided practice
effective classroom manager. What is one sign of “with-it-ness”? b. A review of previous day’s work
a. Giving attention to students who are having difficulty with c. Advance organizers that provide an overall picture of the
school work lesson
b. Aware of what’s happening in all parts of the classroom d. Independent
c. Seeing only a portion of the class but intensively
352. One’s approach to teaching is influenced by Howard Gardner’s MI
d. Knowing where instructional materials are kept
Theory. What is he/she challenged to do?
I. To come up with 9 different ways of approaching lesson to
342. Which of these is one of the ways by which the internet enables
cater to the 9 multiple intelligence
people to browse documents connected by hypertext links?
II. To develop all student’s skill in all nine intelligences
a. URL c. Welcome page III. To provide worthwhile activities that acknowledge individual
b. Browser d. World Wide Web difference in children
343. Which characteristics must be primarily considered as a choice of a. I, II and III c. II only
instructional aides?
b. II and III d. III only
a. Stimulate and maintain students interests
b. Suited to the lesson objectives 353. If my approach to my lesson is behaviorist, what features will
c. Updated and relevant to Filipino setting dominate my lesson?
d. New and skillfully made I. Copying notes III. Lecturing
344. You can exhibit referent power on the first day of school by II. Reasoning IV. Demonstration
__________. a. III, IV c. I, II, III, IV
a. telling them the importance of good grades b. I, III, IV d. II, III, IV
b. giving your students a sense of belongingness and 354. You practice inclusive education. Which of these applies to you?
acceptance I. You accept every student as full and valued member of the
c. making them feel you know what you are talking about class and school community

17
II. Your special attention is on learners with specific learning or Based on this information, which of the following is a valid
social needs conclusion?
III. Your address the needs of the class as a whole within the a. The teacher wants to make her teaching easier by having
context of the learners with specific learning or social needs less talk
a. II only c. I only b. The teacher emphasizing reading and writing skills
b. I and II d. I and III c. The teacher is applying Bloom’s hierarchy of cognitive
355. School curriculum reflects the world’s economic and political learning
integration and industrialization. What does these points in d. The teacher is teaching in a variety of ways because not
curriculum development? all students learn in the same manner
a. The trend towards the classical approach to curriculum
development 364. Teacher Georgina, an experienced teacher, does daily review of
b. The trend towards the globalization and localization past lessons in order to:
c. The trend towards participatory curriculum development a. introduce a new lesson
d. The shift in the paradigm of curriculum development b. reflect on how she presented the previous lessons
from a process-oriented to a product-oriented one c. provide her pupils with a sense of continuity
356. You choose cooperative learning as a teaching approach. What d. determine who among her pupils are studying
thought is impressed on your students?
a. Interaction is a must, but not necessarily face to-face 365. I combined several subject areas in order to focus on a single
interaction concept for interdisciplinary teaching. Which strategy did I use?
b. Student’s success depends on the success of the group a. Reading-writing activity
c. Student’s individuality evaluates how effectively their group b. Thematic introduction
worked c. Unit method
d. The accountability for learning is on the group not on the d. Problem-centered learning
individual
357. Which statement applies correctly to Edgar Dale’s “Cone of 366. To teach the democratic process to the pupils, Batongmalaki
Experience”? Elementary School decided that the election of class officers shall
a. The farther you are from the base, the more direct the be patterned after local elections. There are qualifications set for
learning experience becomes candidates, limited period for campaign and rules for posting
b. The farther you are from the bottom, the more direct the campaign materials, etc. Which of the following did the school
learning experience becomes use?
c. The closer you are from the base, the more indirect the a. Symposium c. Pole playing
learning experience becomes b. Simulation d. Philips 66
d. The closer you are from the base, the more direct the 367. Which among the following are effective methods in teaching
learning experience becomes student critical reading skills?
358. “When more senses are stimulated, teaching and learning become I. Interpret editorial
more effective.” What is an application of this principle? II. Read and interpret three different movie reviews
a. Appeal to student’s sense of imagination III. Read a position paper and deduce underlying assumptions of
b. Use multisensory aids the position papers
c. Make your students touch the instructional material a. II and III c. I and II
d. Use audiovisual aids because the eyes and the ears are the b. I and III d. I, II and III
most important senses in learning 368. Here is a test item:
359. Which is a classroom application of the theory of “operant “The improvement of basic education should be the top priority of
conditioning”? the Philippine government. Defend or refute this position.”
a. Help student see the connectedness of facts, concepts, and Under what type of question does this test item fall?
principles a. Low-level c. Analysis
b. Create a classroom atmosphere that elicits relaxation b. Evaluative d. Convergent
c. Reinforce a good behavior to increase the likelihood that 369. When I teach, I often engage in brainstorming. Which do I avoid?
the learner will repeat the response a. Break down barriers
d. Make students learn by operating manipulatively b. Selectively involves pupils
360. Read the following teacher-student situation. TEACHER: Why is c. Increase creativity
the process called photosynthesis? d. Generate many ideas
STUDENT: I don’t know. 370. Teacher Sammy teaches a lesson in which students must
Which questioning technique should be the teacher be using? recognize that ¼ is the same as 0.25. He use this relationship to
a. Clarification c. Prompting determine that 0.15 and 0.20 are slightly less than ¼. Which of the
b. Multiple response d. Concept review following concept/s is/are being taught?
a. Numeration skills of decimals and relationships between
361. Here is the test item. fractions and decimals
“From the data presented in the table, form generalizations that b. Numeration skills
are supported by the data”. c. Place value of decimals
Under what type of question does this item fall? d. Relationship between fraction and decimals
a. Convergent c. Application 371. What is the best way to develop math concept?
b. Evaluative d. Divergent a. Solving problems using multiple approaches
b. Solving problems by looking for correct answer
362. I want to teach concepts, patterns and abstractions. Which method c. Learning math as applied to situations, such as being a
will be most appropriate? tool of science
a. Discovery c. Direct instruction d. Solving problems by applying learned formulas
b. Indirect instruction d. Problem solving 372. After the reading of a selection in the class, which of these
activities can enhance students’ creativity?
363. Teacher Atilla teaches English as a Second Language. She uses I. Reader’s theater
vocabulary cards, fill-in-the-blanks sentences, dialogues, dictation II. Reading aloud
and writing exercises in teaching a lesson about grocery shopping. III. Silent reading
18
a. I and II c. I only b. Generalization d. Discrimination
b. II only d. III only 384. Based on Freud’s theory, which operate/s when a student strikes a
373. Teacher Carlo, a Reading teacher, advised the class to “read classmate at the height of anger?
between the lines”. What does she want his pupils to do? a. Ego c. Id and Ego interact
a. Determine what is meant by what is stated b. Id d. Superego
b. Make an educated guess 385. After reading an essay. Teacher Beatrice wants to help sharpen
c. Apply the information being read her students’ ability to interpret. Which of these activities will be
d. Describe the characters in the story most appropriate?
374. To nurture students’ creativity, which activity should a teacher a. Drawing conclusions
avoid? b. Making inferences
a. Ask “What if…” questions c. Getting the main idea
b. Emphasize the need to give right answers d. Listing facts separately from opinion
c. Ask divergent thinking questions 386. Read the following then answer the question. A man and his son
d. Be open to “out-of-this-world” ideas are driving in a car. The car crashes into a tree, killing the father
375. Teacher Rodel wants to develop his students’ creativity. Which and seriously injury his son. At the hospital, the boy needs to have
type of questions will be most appropriate? surgery. Looking at the boy, the doctor says (telling the truth), “I
a. Synthesis questions cannot operate on him. He is my son. How can this be?
b. Fact questions ANSWER: The doctor is the boy’s mother.
c. “What if…” questions The above brain twister helps develop critical reading skills. Which
d. Analysis questions activity was used?
376. Floramay enjoyed the roller coaster when she and her family went a. Comparing c. Inferring meaning
to Disneyland. The mere sight of a roller coaster gets her excited. b. Classifying d. Looking for cause and effect
Which theory explains Floramay’s behavior? 387. Research says that mastery experiences increase confidence and
a. Operant conditioning willingness to try similar or more challenging tasks as reading.
b. Pavlovian conditioning What does this imply for children’s reading performance?
c. Social learning theory a. Children who have not mastered the basic skills are more
likely to be motivated to read in order to gain mastery over
d. Attribution theory
basic skills
377. According to Freud, with which should one be concerned if he/she
has to develop in the students a correct sense of right and wrong? b. Children who have mastered basic skills are more likely to be
I. Super-ego II. Ego III. Id less motivated to read because they get fed up with too much
reading
a. I and II c. I
c. Children who have gained mastery over basic skills are
b. II d. III
more motivated to read
378. When small children call animals “dog”, what process is illustrated
on Piaget’s cognitive development theory? d. Children who have a high sense of self-confidence are not
necessarily those who can read
a. Reversion c. Accommodation
388. The value that students put on reading is critical to their success.
b. Assimilation d. Conservation
In what way/s can teachers inculcate his value for reading?
379. Researchers found that when a child is engaged in a learning
I. Sharing the excitement of read-aloud
experience a number of areas of the brain are simultaneously
II. Showing their passion for reading
activated. Which of the following is/are implication/s of this
III. Being rewarded to demonstrate the value of reading
research finding?
a. II and III c. I, II and III
I. Make use of field trips and guest speakers
II. Do multicultural units of study b. I and II d. II only
III. Stick to the “left brain and right brain” approach 389. Bruner’s theory on intellectual development moves from enactive
to iconic and symbolic stages. Applying Bruner’s theory. How
a. I and III c. I and II
would you teach?
b. I only d. II only
a. Be interactive in approach
380. My problem is there are too many topics to cover and I may not
able to finish before classes end in March. Which approach when b. Begin with the abstract
used can help solve my problem? c. Begin with the concrete
a. Thematic c. Experimental approach d. Do direct instruction
b. Constructive d. Direct instruction 390. A person who has painful experiences at the dentist’s office may
381. You want your students to answer the questions at the end of a become fearful at the mere sight of the dentist’s office building.
reading lesson. “What did I learn?”, “What still puzzles me?”, Which theory can explain this?
“What did I enjoy, hate and accomplish in the class today?” and a. Generalization
“How did I learn from the lesson?”.Which of the following are you b. Operant Conditioning
asking them to do? c. Attribution theory
a. Work on an assignment d. Classical conditioning
b. Make journal entry 391. Which is/are the basic assumption/s of behaviorists?
c. Work on a drill I. The mind of newborn child is a blank state
d. Apply what they learned II. All behaviors are determined by environmental events
382. William Glasser’s control theory states that behavior is inspired by III. The child has a certain degree of freedom not to allow himself
what satisfies a person want at any given time. What then must a to be shaped by his environment
teacher do to motivate students to learn? a. III only c. II only
a. Make schoolwork relevant to students’ basic human b. I and II d. I and III
needs 392. If a student is encourage to develop himself to the fullest and must
b. Make teaching-leaning interactive satisfy his hierarchy of needs, the highest needs to satisfy
c. Avoid giving assignments according to Maslow is ________.
d. Organize a curriculum in a spiral manner a. Psychological need c. Belongingness
383. Nadia exhibit fear response to freely roaming dogs but does not b. Self-actualization d. Safety needs
show fear when a dog is on a leash or confined to a pen. Which 393. In a Social Studies class. Teacher Ina presents a morally
conditioning process is illustrated? ambiguous situation and asks student what they would do. On
a. Extinction c. Acquisition whose theory is Teacher Ina’s technique based?
19
a. Bandura c. Kohlberg a. Tell them the research finding when applied will make them
b. Piaget d. Bruner genuinely motivated
394. Teacher Violy is convinced that whenever a student performs a b. Convince them that genuine motivation is the only factor
desired behavior, provide reinforcement and soon the student that matters for a person to succeed
learns to perform the behavior on his own. On which principle is c. Make them realize that failure is a part of life
Teacher Violy’s conviction based? d. Make them realize that both success and failure are more a
a. Environmentalism c. Cognitivism function of internal causes
b. Behaviorism d. Constructivism
395. In Bandura’s social learning theory, it states that children often 403. Which characterize/s a learning environment that promotes
imitate those who: fairness among learners of various cultures, family background
I. have substantial influence over their lives and gender?
II. belong their peer group I. Inclusive III. Gender-sensitive
III. belong to other race II. Exclusive
IV. are successful and seem admired a. I only c. I and III
a. IV only c. I and II b. III only d. II and III
b. I and IV d. II and IV
396. According to Erikson, what years are critical for the development of 404. Which of the following steps should be completed first in
self-confidence? planning an achievement test?
a. High school years a. Define the instructional objective
b. Elementary school years b. Set up a table of specialization
c. College years c. Select the types of test items to use
d. Preschool years d. Decide on the length of the test
397. Which of the following does not describe the development of 405. The computed r for scores in Math and Science is 0.92. What
children aged 11 to 13? does this mean?
a. They exhibit increased objectivity in thinking a. Math score is positive related to Science score
b. Sex difference in IQ become more evident b. The higher the Math score, the lower the Science score
c. They shift from impulsivity to adaptive ability c. Math score is not in any way related to Science score
d. They show abstract thinking and judgment d. Science score is slightly related to Math score
398. Teacher Henry begins a lesson on tumbling, demonstrating front
and back somersaults in slow motion and physically guiding his 406. Which types of test is most appropriate if Teacher Yanny wants
students through the correct movements. As his students become to measure student’s ability to organize thoughts and ideas?
more skillful, he stands back from the man and gives verbal a. Short answer type of test
feedback about how to improve. With Vygotsky’s theory in mind, b. Extended response essay
what did Teacher Henry do? c. Modified alternative response
a. Apprenticeship c. Peer interaction d. Limited response essay
b. Guided participation d. Scaffolding
399. What does Gagne’s hierarchy theory propose for effective 407. If I want to hone my student’s meta-cognitive ability, which is
instruction? most fit?
a. Be concerned with the socio-emotional climate in the a. Drill c. Brainstorming
classroom b. Debate d. Journaling
b. Sequence instruction
c. Teach beginning with the concrete 408. I want to test student’s synthesizing skills. Which has the
d. Reward good behavior highest diagnostic value?
400. Which appropriate teaching practice flows from this research a. Multiple choice test
finding on the brain: “The brain’s emotional center is tied into its b. Performance test
ability to learn”. c. Essay test
a. Establish the discipline of being judgmental in attitude d. Completion test
b. Come up with highly competitive games where winners will
feel happy 409. Here is a test item:
c. Create a learning environment that encourages
_________ is an example of a
students to explore their feeling and ideas freely
d. Tell the students to participate in class activities or else leafy vegetable.
won’t receive plus points in class recitation
Why is this test item poor?
401. Research on Piagetian tasks indicates that thinking becomes I. The test item does not pose a problem to the
more logical and abstract as children reach the formal operation examinee
stage. What is an educational implication of this finding? II. There are variety of possible correct answer to this item
a. Engage children in analogical reasoning as early as III. The language used in the question is not precise
preschool to train them for higher order thinking skills IV. The blank is near the beginning of a sentence
(HOTS) a. I and III c. I and IV
b. Learners who are not capable of logical reasoning from b. II and IV d. I and II
ages 8 to 11 lag behind in their cognitive development
c. Let children be children 410. Joanne’s score is within x±1 SD. To which of the following
d. Expect hypothetical reasoning for learners between 12 groups does she belong?
to 15 years of age a. Below average
b. Average
402. Research says: “People tend to attribute their successes to c. Needs Improvement
internal causes and their failures to external causes.” Based on d. Above average
this finding, what should be taught to students for them to be 411. Here is a test item:
genuinely motivated to succeed?
What follows is a multiple choice type of
test. Some test items _________.
20
a. are too difficult
b. are objective
c. are poorly constructed
c. Percentile Rank
d. Percentage Score
419. Use the inbox below to answer the question that follows:
Percentage Grades for Final Examination

What makes the multiple choice type of test poor? 40 70 80 90 100


a. The options are not grammatically connected to the stem Which of the following statement is true about the plot of grades
b. The stem fails to present a problem above?
c. There are grammatical clues a. The median is a score of 80 and the range is 60
d. The options are not parallel b. The median is a score of 70 and the range is 60
412. If Teacher Betty wants to measure her students’ ability to c. The median is a score of 80 and the range is 20
discriminate, which of these is an appropriate type of test item d. The median is a score of 70 and the range is 20
as implied by the direction? 420. Which can be said of Nina who obtained a score of 75 out of
a. “Outline the Chapter on The Cell.” 100 items in a Grammar objective test?
b. “Summarize the lesson yesterday.” a. She performed better than 25% of her classmates
c. “Group the following items according to shape.” b. She answered 75 items in the test correctly
d. “State a set of principle that can explain the following c. Her rating is 75
events.” d. She answered 75% of the test items correctly
421. The criterion of success in Teacher Butch’s objective is that “the
413. A test item has a difficult index of 0.89 and a discrimination pupils must be able to spell 90% of the words correctly”. Student
index of 0.44. What should the teacher do? Dave and 24 others in the class spelled only 40 out of 50 words
a. Reject the item correctly while the rest scored 45 and above. This means that
b. Revise the item Teacher Butch _____________.
c. Make it a bonus item a. attained his lesson objective
d. Make it a bonus item and reject it b. failed to attain his lesson objective as far as the 25
pupils are concerned
414. Which form of assessment is consistent with the saying “The c. did not attain his lesson objective because of the pupils’
proof of the pudding is in the eating.” lack of attention
a. Contrived c. Traditional d. attained his lesson objective because of his effective
b. Authentic d. Indirect spelling drill
422. Which of these can measure awareness of values?
415. Here is a test item: a. Sociogram
b. Moral dilemmas
Who is best admired for outstanding c. Projective techniques
contribution to world peace? d. Rating scales
423. Marking on a normative basis means that:
a. Kissinger c. Kennedy a. the normal distribution curve should be followed
b. Clinton d. Mother Teresa b. some should fall
c. some get high marks
d. the grading is based on a present criteria
What is WRONG with this item?
424. Which process enhances the comparability of grades?
a. Item is overly specific
a. Using a table specifications
b. Content is trivial
b. Determining the level of difficulty of the tests
c. Test item is option-based
c. Giving more HOTS (higher order thinking skills)
d. There is a cue to the right answer
d. Constructing departmentalized exams for each subject
area.
416. A student’s score were as follows: 82, 83, 84, 86, 88, 84, 83, 85.
425. If the scores of your test follow a negatively skewed score
The score 84 is the:
distribution, what should you do? Find out __________?
a. Mode c. Median
a. why your items were easy
b. Average d. Mean
b. why most of the scores are high
c. why most of the scores are low
417. Which text form would you choose if you want to have a valid
and reliable test based on the table below? d. why some pupils scored high
426. Principal Gemma is talking about “grading on the curve” in a
Test Form Validity Index Reliability Index faculty meeting. What does this expression refers to?
a. A student’s mark compares his achievement to his effort
A .47 .68 b. A student’s grade or mark depends on how his
B .87 .57 achievement compares with the achievement of other
C .20 .86 students in a class
D .40 .41 c. A student’s grade determines whether or not a student
E .63 .07 attains a defined standard of achievement
a. A only c. A and D d. A student’s mark tells how closely he is achieving to his
potential
b. B only d. B and E
427. Which tests determine whether students accept responsibility for
418. A mathematician’s test was given to all Grade V pupils to
their own behavior or pass on responsibility for their own
determine the contestants for the Math Quiz Bee. Which
behavior to other people?
statistical measure should be used to identify the top 15?
a. Locus-of-control tests
a. Mean percentage score
b. Thematic tests
b. Quartile Deviation
21
c. Stylistic test
d. Sentence-completion tests

428. Which of the following is a weakness of self-supporting


personality checklists?
a. Many personality measures have built-in lie scales
b. They lack stability
c. They may not get true information because individuals
can hide or disguise feelings
d. They have poor internal consistency

Situation 1. (#129-131) 435. The class was asked to share their insights about the poem.
The ability to come up with an insight stems from the ability to:
In a faculty meeting, the principle told a. analyze the parts of a whole
his teacher: “We need to improve our b. evaluate the worthiness of a thing
c. relate and organize things and ideas
school performance in the National
d. comprehend the subject that is being studied
Achievement Test. What should we do?” 436. To ask the class any insight derived from the poem is based on
the theory of:
The teacher gave varied answers as a. Realism c. Conditioning
follows: b. Behaviorism d. Constructivism
437. On which assumption about the learner is Mr. Sales’s act of
1. Let’s give incentives and rewards to asking the class to share their insight based?
students who get a rating of 85%. a. Learners are like empty receptacles waiting to be filled up
2. Let’s teach them to accept complete b. Learners are meant to interact with one another
responsibility for their performance. c. Learners have multiple intelligence and varied learning
styles
3. Let’s make the school environment d. Learners are producers of knowledge not only passive
conducive for learning. recipients of information
4. Let’s make use of the experimental
Situation 4. (#138-140)
methods of teaching.
Principal Evelyn wants her teachers to
429. Which response/s come/s from a behaviorist? apply constructivism in teaching.
a. #2 and #4 c. #3 and #4
b. #1 and #2 d. #1 and #3
438. On which assumption/s is the principal’s action anchored?
430. On which educational philosophy is response #1 anchored? I. Students learn by personally constructing meaning
a. Existentialism c. Progressivism of what is taught
b. Essentialism d. Bahaviorism II. Students construct and reconstruct meaning based on
experiences
431. If you leaned toward a progressivist philosophy, with which III. Students derive meaning from the meaning that the
response would you agree? teacher gives
a. #2 b. #3 c. #4 d. #1 a. II only c. I, II and III
b. I and II d. I only
Situation 2. (#132-134)
439. Which materials will her teachers least prefers?
One principle in the utilization of
a. Controversial issues
technology of the classroom is b. Open-ended topics
appropriateness of material or activity. c. Unquestionable laws
d. Problem or cases
440. Which concept/s of the learner will Principal Evelyn not accept?
432. Teacher Wilson wants his students to master the concept of I. “Empty vesse!”
social justice. Which series of activities will be most effective? II. “Tabula rasa”
a. Pretest-teaching-posttest III. Candle to be lighted
b. Pretest-teaching-posttest-re-teaching for unlearned a. III only c. II only
concepts-posttest b. I only d. I and II
c. Review-pretest-teaching-posttest
d. Teaching-posttest
433. Teacher Raymund likes to show how the launching of
spaceships takes place. Which of the following materials
available is most fit?
a. Model b. Mock-up c. Replica d. Realia
434. Teacher June likes to concretize the abstract concepts of an
atom. He came up with a concrete presentation of the atom by
using wire and plastic balls. How would you classify Teacher
June’s visual aids?
a. Chart b. Replica c. Model d. Realia Situation 5. (#141-143)

Situation 3. (#135-137) Study the matching type of test then answer


the 3 questions that follow:
After reading and paraphrasing Robert
Frost’s “Stopping by the Wood on a 22 Column A Column B
Snowy Evening.” Mr. Sales asked the
class to share any insight derived from 1. equilateral triangle A. With 3 equal
sides
the poem.
2. right triangle B. With 5 equal
II. It has a rating scheme
III. It limit itself to 4 levels of achievement
a. I and II c. II and III
b. I and III d. I, II and III
447. Which guidelines should you follow for behavior modification to
be effective?
a. Subject yourself to group pressure
b. Work on several behaviors at the time for a significant
change
c. Never use negative reinforcement
d. Work on one behavior at a time
448. In which phases/s of change, according to Kurt Lewin, do you
feel the pain?
a. Transition c. Refreezing
b. Unfreezing d. Transition and freezing
441. How can you make the items homogeneous? 449. In the writing of performance objectives for mastery learning,
a. Increase the number of items in Column B what is an acceptable standard of performance called?
b. All items should be on polygons a. Behavior c. SMART
c. Remove the word triangle in items #1 and #2 in column A b. Condition d. Criterion measure
d. The word “gon” must be included in column B 450. Why learning activities should be carefully planned?
442. What is the main defect of this matching test? a. The ability of the teacher to plan is tested
a. The matching type is an imperfect type b. This is expected by pupils
b. The items are NOT homogeneous c. The accomplishment of objectives is dependent on the
c. The items quite easy plan
d. An obvious pattern is followed in the answering d. This is required of a teacher
443. Which should be done to improve the matching type of test?
a. Capitalize the items in Column A 451. Which statement on IQ and EQ is correct?
a. EQ has a greater contribution to performance than IQ
b. Items in Column A and B should be exchanged
b. IQ has a greater contribution to performance than EQ
c. Drop #6 item in Column A
c. Blending of both IQ and EQ can make a difference in
d. The item in Column A should be increased performance
d. The contribution of IQ and EQ to performance is
dependent on factors like age and gender
Situation 6. (#144-146)
452. Teacher Nikko helped his students recall that stalagmites grow on
Below the template for Scoring Rubric.
the “ground” while stalactites grow on the “ceiling” of a cave by
associating “G” in stalaGmites with ground and “C” in stalaCtites
with ceiling. What did Teacher Nikko make use of it?
5 - Demonstrate complete understanding of the problem. All a. Visual aid
requirements of task are included in the response b. Mnemonic device
c. Audio-visual aid
d. Meaning-maker device
4 - Demonstrate considerable understanding of the problem. All
453. “Makabayan” as a subject in the re-structured Basic Education
requirements of task are included Curriculum is the “laboratory for life.” What does this mean? It is in
this subject where the learner ____________.
3 - Demonstrate partial understanding of the problem. Most a. Will be taught the Filipino strengths and weaknesses
requirements of task are included b. Will demonstrate practical knowledge and skills
gained in the other subjects
c. Will be taught the true concept of being
2 - Demonstrate little understanding of the problem. Many
“pagkamakabayan”
requirements of task are missing d. The biographies of heroes who are “makabayan” will be
taught
1 - Demonstrate no understanding of the problem 454. In the faculty room everyone is talking about a teacher who is
torturing for a fee from her own pupil who is vying for honors. What
0 - No response/task not attempted is the professional thing for the other teachers to do?
a. Talk to the parents of the tutee. Tell them what teachers
doing is unprofessional
444. Which of this/these is/are essential in constructing a scoring b. Leave her alone, she might accuse you of meddling in
rubric? her personal life
I. Description of criteria to serve as standard c. Correct her and remind her torturing one’s own pupil
II. Clear descriptions of performance at each level for a fee is unethical
III. Levels of achievement (mastery) d. As a group, report her to the principal
IV. Rating scheme 455. When Teacher Demi presents a set of data then asks the students
a. I, II and III c. I, II, III and IV to enter a conclusion, generalization or a pattern of relationship
b. I and II d. I only which method does she use?
445. Which statement is true of the rubric? a. Process approach
a. It is developmental b. Type method
b. It is analytical c. Unit method
c. It is both holistic and developmental d. Inductive inquiry method
d. It is holistic 456. Which individualized teaching method makes use of workbooks,
446. Which is true of the scoring rubric? teaching machines or computers?
I. It describes criteria of levels of achievement a. Project method

23
b. Unit method b. are a measure of achievement
c. Programmed instruction c. are necessarily a measure of students’ IQ
d. Inductive inquiry method d. are intrinsic motivation for learning
457. I want to elicit more student responses. Which one should I avoid? 469. Which practice does not fit in a classroom that recognizes
a. Creating an evaluative atmosphere individual differences?
b. Using covert responses a. Uniform requirements
c. Prompting to covert wrong answers to correct ones b. Sharing from multiple perspective
d. Giving non-threatening comments c. Accommodating student’s learning styles
458. For practice to be effective, which guideline should bear in mind? d. Various modes of assessing learning
Practice should _______. 470. Teacher Benny says: “If it is billiard that brings students out of the
a. Be done in an evaluative atmosphere classroom, let us bring it into the classroom. Perhaps, I can use it
b. Be arranged to allow students to receive feedback to teach Math.” To which philosophy does Teacher Benny adhere?
c. Take place over a long period of time a. Progressivism c. Essentialism
d. Be difficult enough for students to remember and learn b. Existentialism d. Reconstructionism
459. Which of the following is a practice in progressive education? 471. “The greatest happiness lies in the contemplative use of the mind”,
a. Open classroom said Plato. Therefore, let us give more opportunities for our
b. Study of classics students to do __________.
c. Academic orientation a. Social interaction c. Role playing
d. Cognitive education b. Introspection d. Cooperative learning
460. The workers’ rights to form unions or to strike can be suppressed 472. Why is it sound to encourage students to define terms in their own
in times of national emergency. On what norm is this based? words? Because ____________.
a. Higher law, inalienable rights before alienable a. Defining the terms in their own words helps them
b. Wider social order, the society before the individual memorize the definition faster
c. Clearer title, the certain before the title b. Students remember information better when they
d. Nobler person, God before man mentally process in some way
461. Which skills should be taught if Teacher Joey wants to equip his c. They ought to connect the terms that they learn with
students with the skill to organize information gathered? other terms
a. Note-taking, outlining, using the library d. This is one opportunity to brush up with other terms
b. Outlining, summarizing, using the card catalogue 473. Pavlov is to classical conditioning as ______ is to operant
c. Note-taking, outlining, summarizing conditioning.
d. Summarizing, note taking using the library a. A. Bandura c. J. Watson
462. Annual medical checkup required of teachers is done in the b. J. Holt d. B.F. Skinner
interest of: 474. You arrange the rows of blocks in such a way that a row of 5
a. Filipino medical doctors blocks is longer than a row of 7 blocks. If you ask which row has
b. Parents more, Grade 1 pupils will likely say that it is the row that makes the
c. The state and of every teacher longer line. Based on Piaget’s cognitive development theory, what
d. School administration problem is illustrated?
463. In a study conducted, the pupils were asked which nationality they a. Conservation problem
preferred if given a choice. Majority of the pupils wanted to be an b. Assimilation problem
American. In this case, in which obligation relative to the state are c. Egocentrism problem
schools seemed to be failing? In their obligation to: d. Accommodation problem
a. Respect for all duly constituted authorities 475. Were teachers in the Philippines required of a professional license
b. Instill allegiance to the Constitution since the establishment of the Philippine educational system?
c. Promote obedience to the laws of the state a. No, but the equivalent of a license required was a
d. Promote national pride certificate in teaching
464. To be an effective classroom manager, teachers must be friendly b. Yes, it was required since the Americans established the
but at the same time be: educational system
a. Buddy-buddy c. No, it was only with the effectivity of R.A. 7836 that
b. Rigid professional license was required
c. Business-like d. Yes, except for the Thomasites
d. Highly demanding 476. The increase in the number of school children left by OFW parents
465. Which of the following field of Social Sciences below is more intensifies the teacher role as ______________.
connected with the study of social traditions and cultures? a. Student’s friends
a. Theology b. Guidance counselors
b. Psychology c. Facilitator of learning
c. Sociology d. Substitute parents
d. Anthropology 477. As a teacher, you are a rationalist. Which among these will be your
466. When an individual or group adapts the culture of others, practice guiding principle?
them and become habitual, this is: a. I must teach the child so he is assured of heaven
a. Culture change c. Culture shock b. I must teach the child to develop his mental powers
b. Culture lag d. Culture difference to the fullest
467. Which appropriate teaching practice flows this research finding on c. I must teach the child that we can never have real
the brain: “The brain’s emotional center is tied into its ability to knowledge of anything
learn.” d. I must teach the child every knowledge, skill and value
a. Tell the students to participate in class activities or else that needs for a better future
won’t receive plus points 478. A teacher put together the output of her colleagues in one
b. Create a learning environment that encourages workshop and published it with her name as author. Which is
students to explore their feelings and ideas unprofessional about the teacher’s behavior?
c. Come up with highly competitive games where winners a. Failing to correct what appears to be unprofessional
will feel happy conduct
d. Establish the disciple of being judgmental in attitude b. Giving due credit to others of their work
468. Which statement holds true to grades? Grades _________. c. Not giving due credit to others for their work
a. are exact measure of IQ and achievement
24
d. Holding inviolate all confidential information concerning 489. Teacher Janice observes cleanliness and order in her classroom
associates to create a conductive atmosphere for learning. On which theory is
479. In what way can teachers uphold the highest possible standards of her practice based?
the teaching profession? a. Behaviorism
a. By pointing out the advantages of joining the teaching b. Psychoanalysis
profession c. Gestalt psychology
b. By good grooming to change people’s perception of d. Humanistic psychology
teacher 490. Which activity is meant for kinesthetically intelligent pupils?
c. By continuously improving themselves personally a. Independent study
and professionally b. Individualized study
d. None of the above c. Pantomime
480. To reach out to clientele who cannot be in the classroom for one d. Cooperative learning
reason or another, which of the following was established? 491. With which will the existentialist agree? The school is a place
a. Special education (SPED) where individuals _____.
b. Informal education a. Listen and accept what the teacher say
c. Alternative learning delivery system b. Can meet to pursue dialogue and discussion about
d. Pre-school education their lives and choices
481. Teacher Neil discovered that his pupils are weak in c. Can observe by using their senses to the maximum
comprehension. To further determine in which particular skills his d. Can reflect on ideas
pupils are weak; which test should Teacher Neil give? 492. Which will be the most authentic assessment tool for an
a. Standard Test c. Diagnostic Test instructional objective on working with and relating to people?
b. Placement Test d. Aptitude Test a. Writing articles on working and relating to people
482. In the context of multiple intelligences, which one is the weakness b. Organizing a community project
of the paper-pencil test? c. Home visitation
a. It put non-linguistically intelligent pupils at a d. Conducting mock election
disadvantage
b. It requires paper and printing and is so expensive 493. Which physical arrangement of chairs contributes to effective
c. It utilizes so much time classroom management?
d. It lacks reliability a. Sticks to the traditional chair arrangement in the
483. Out of 3 distracters in a multiple choice test item, namely X, Y and classroom
Z, no pupil chose Z as an answer. This implies that Z is ________. b. Distinguishes teacher from students
a. An effective distracter c. Makes it easier to clean the room
b. A plausible distracter d. Enhances classroom interaction
c. A vague distracter
d. An ineffective distracter 494. Each teacher said to be a trustee of cultural and educational
heritage of the nation and under obligation to transmit to learners
484. The Thematic Appreciation Test is an example of a (an) such heritage. Which practice makes him fulfill such obligation?
__________. a. Use of the latest instructional technology
a. Self-report technique b. Study of the life of Filipino heroes
b. Projective technique c. Use of interactive teaching strategies
c. Interest inventory d. Observing continuing professional education
d. Socio-metric technique 495. Writing an original essay is an example of which level of objective
in the cognitive domain?
485. The following are features of the Restructure Basic Education a. Evaluation c. Analysis
Curriculum, except: b. Synthesis d. Application
a. Increased time for tasks to gain mastery of
competencies 496. Which terms refers to a teacher helping a colleague grow
b. Interdisciplinary modes of teaching professionally?
c. Greater emphasis on content, less on the learning a. Technology transfer
process b. Independent study
d. Stronger integration of competencies and values, across c. Facilitating
the learning area d. Peer mentoring
497. Which terms refers to the collection of student’s products and
486. The free public elementary and secondary education in the country accomplishments for a period for evaluation purposes?
is in the line with the government effort to address educational a. Anecdotal record c. Diary
problems of _______. b. Observation report d. Portfolio
a. access and equity 498. To develop reasoning and speaking ability, which should I use?
b. relevance and quality a. Debate c. Experiment
c. effectiveness and efficiency b. Storytelling d. Role-playing
d. productivity 499. Under which type of guidance service does the concern of schools
487. The task of setting up routine activities for effective classroom to put students into their most appropriate courses fall?
management as a task that a teacher should undertake ______. a. Individual inventory service
a. on the very first day of school b. Research service
b. every day at the start of the session c. Placement service
c. every homeroom day d. Information service
d. as soon as the students have adjusted on their schedule 500. After having been humiliated by his teacher, a student evaluates
488. Teacher Honey uses direct instruction strategy. Which will she first that teacher very poorly, despite teacher’s excellent performance.
do? Which trait is illustrated by the student’s behavior?
a. Independent practice a. Particularism c. Personalism
b. Guided student practice b. Rationalism d. Impersonalism
c. Review the previous day’s work
d. Presenting and structuring 501. Conducting follow up studies of graduates and drop out is a
guidance service that falls under:
25
a. Placement services a. Symposium c. Brainstorming
b. Research service b. Debate d. Panel discussion
c. Individual inventory services 512. The criterion of success in Teacher Edna’s objective is that “the
d. Counseling service pupils must be able to spell 90% of the words correctly.” Linda and
other 24 students in the class spelled only 40 out of 50 words
correctly while the rest scored 45 and above. This means that
502. Under which assumption is portfolio assessment based? Teacher Edna ________.
a. Assessment should stress the reproduction of a. Did not attain her lesson objective because of the pupil’s
knowledge lack of attention
b. An individual learner is adequately characterized by a b. Attained her lesson objective because of the pupil’s lack
test score of attention
c. An individual learner is inadequately characterized by a c. Attained her lesson objective
test score d. Failed to attain her lesson objective as far as the 25
d. Portfolio assessment is dynamic assessment pupils are concerned
503. Which program was adopted to provide universal access to basic 513. With assessment of affective learning in mind, which does not
education to eradicate illiteracy? belong to the group?
a. Values educational framework a. Moral dilemma c. Diary entry
b. Education for all b. Reflective writing d. Cloze test
c. “Paaralan sa bawat barangay” 514. Which is true when standard deviation is big?
d. Science and Education Development Plan a. Scores are concentrated
504. Which schools are subject to supervision, regulation and control by b. Scores are not extremes
the state? c. Scores are spread apart
a. Public, Private sectarian and Non-sectarian d. The bell curve shape is steep
b. Sectarian and non-sectarian school 515. Jan, a grade 1 pupil, is happy when he wins a game but skulks
c. Private school when he doesn’t. Which concept does his behavior indicate?
d. Public schools a. Egotism c. Semi-logical reasoning
505. Here is a score distribution: 98, 93, 93, 93, 90, 88, 87, 85, 85, 85, b. Egocentrism d. Rigidity of thought
70, 51, 34, 34, 34, 20, 18, 15, 12, 9, 8, 6, 3, 1. What is the 516. The practice of non-graded instruction stems from ________.
characteristic of the score distribution? a. Progressivism c. Existentialism
a. Bimodal b. Reconstructionism d. Essentialism
b. Trimodal 517. Theft of school equipment like TV, computer, etc. by teenagers in
c. Skewed to the right the community itself is becoming a common phenomenon. What
d. No discernible pattern does this incident signify?
506. What do the school campus expression “promdi” and barriotic” a. Deprivation of Filipino schools
indicate? b. Inability of school to hire security guards
a. The powerlessness of the poor c. Prevalence of poverty in the community
b. The power of the rich d. Community’s lack of sense of co-ownership
c. Low literacy rate of the country 518. What does extreme authoritarianism in the home reinforce in
d. The prevalence of ethnocentrism learners?
507. Why is babyhood referred to as a “critical period” in personally a. Creativity in work
development? Because _________. b. Ability to direct themselves
a. The foundation is laid upon in which the adult c. Doing things on their own initiative
personally structure will be built d. Dependence on others for direction
b. The baby is exposed to many physical and psychological 519. The main purpose of compulsory study of the constitution is to
hazards ________.
c. The brain grows and develops as such accelerated rate a. Make constitutional experts of the students
during babyhood b. Develop students into responsible, thinking citizens
d. Changes in the personality pattern take place c. Prepare students for law-making
508. Billy, a grade 1 pupil is asked, “Why do you pray every day?” Billy d. Acquaint students with the historical development of the
answer. “Mommy said so.” Based on Kohlberg’s theory, in which Philippine Constitution
moral development stage is Billy? 520. Studies in the areas of neurosciences disclosed that the human
a. Pre-Conventional level brain has limitless capacity. What does this imply?
b. Conventional level a. Every child is a potential genius
c. Between conventional and post conventional levels b. Pupils can possibly reach a point where they have
d. Post-Conventional level learned everything
509. If you plan to develop a lesson on using s-verb with the third c. Some pupils are admitted not capable of learning
person singular as subject deductively, what is the first step in your d. Every pupil has his its own native ability and his learning
lesson development outline? is limited to this native ability
a. Give sentences using s-verb form 521. The principle of individual differences requires teachers to _____.
b. Ask the students about s-verb form and third person a. Treat all learners alike while in the classroom
singular as subject b. Prepare modules for slow learners in class
c. State the rule on subject-verb agreement for third c. Give greater attention to gifted learners
person as subject d. Provide for a variety of learning activities
d. Conduct appropriate sentence drill 522. Which assumption underlines the teacher’s use of performance
510. For lesson clarity and effective retention, which one should a objectives?
teacher observes, according to Bruner’s theory? a. Performance objectives assure the learner of learning
a. Start at the concrete level and end there b. Learning is defined as a change in the learner’s
b. Begin teaching at the concrete level but go beyond it observable performance
by reaching the abstract c. The success of learners is based on teacher
c. End teaching with verbal symbol performance
d. Use purely verbal symbols in teaching d. Not every form of learning is observable
511. Which activity should a teacher have more for his students if he 523. The following are used in writing performance objective, except?
wants them to develop logical-critical thinking? a. Integrate c. Diagram
26
b. Delineate d. Comprehend a. Interactive learning
524. Which is/are sign/s of the student with Attention Deficit Disorder? b. Increase learning
a. Impatient while waiting for his/her turn during games c. Speed learning
b. Completes work before shifting to another d. More interesting learning
c. Excessively quiet 536. Which refers to the Filipino trait of practicing conflicting values in
d. Cares for his/her personal things different venues and with different social groups?
525. You observe that pupils answer even when not called, shouts a. “Kanya-kanya” mentality
MA’AM to get your attention, and laugh when someone commits b. Procrastination
mistakes. What should you do? c. Existential intelligence
a. Send the misbehaving pupils to the guidance counselor d. Crab mentality
b. Set the rules for the class to observe 537. Which of these can measure awareness of values?
c. Involve the whole class in setting rules of conduct a. Projective techniques
for the whole class b. Rating scales
d. Make a report to the parents about their children’s c. Moral dilemmas
misbehavior d. Sociogram
526. Which is the final, indispensable component of a lesson plan? 538. Which test determines whether students accept responsibility for
a. Evaluation c. References their own behavior or pass on responsibility for their own behavior
b. Activity d. Assignments to other people?
527. Carlo, a grade 2 pupil, plays with his classmates but cannot accept a. Locus-of-control tests
defeat. Based on Piaget’s theory on cognitive development, in b. Sentence-completion tests
what development stage is Carlo? c. Thematic tests
a. Formal operation c. Pre-operational d. Stylistic tests
b. Concrete operation d. Sensorimotor
528. Which seating arrangement has been proven to be effective for 539. Which process enhances the comparability of grades?
learning? a. Giving more Higher Order thinking skills
a. Flexible to suit varied activities b. Constructing departmentalized exam for each
b. Fixed arrangement to maximize instructional time subject area
c. Any seat arrangement to suit varied learning styles c. Determining the level of difficulty of the test
d. A combination of fixed and flexible arrangement d. Using a table of specification
529. Which can run counter to the encouragement you give to your
students to ask questions? 540. Under which type of guidance service does the concern of school
a. Eye to eye contact to put students into their most appropriate courses fall?
b. An encouraging hand gesture a. Information service
c. Radiant face b. Placement service
d. Knitted eyebrows when a question is raised c. Individual inventory services
d. Research services
530. Teacher Agot likes to show how the launching of spaceships takes 541. The teacher’s role in the classroom according to cognitive
place. Which of the following materials available is most fit? psychologist is to _______.
a. Mock-up c. Replica a. Make the learning task easy for the learner
b. Realia d. Chart b. Dictate what to learn upon the learner
c. Fill the minds of the learner with information
531. Teacher Bonnie likes to concretize abstract concept of the water d. Help the learner connect what they know with new
molecule. She came up with a concrete presentation by using information from the teacher
wires and plastic balls. How would you classify Teacher Bonnie’s
visual aid? 542. Which questioning technique would be appropriate for inductive
a. Replica c. Realia lessons?
b. Chart d. Mock-up a. Involve students actively in the questioning process
532. Teacher Lenny demonstrated to the class how to focus the b. Expect participation only among the more motivated
microscope, after which the students were asked to practice. students
Which teacher prompting is least intrusive? Teacher Lenny c. Use questions requiring only memory responses
________. d. As a teacher, you ask no questions
a. Held the hand of a student and with her hand holding the 543. Which statement is true in a bell-shaped curve?
student’s hand adjusted the mirror a. There are more high scores than low scores
b. Pointed to the mirror and made an adjusting gesture with her b. Most scores are high
hand c. The scores are normally distributed
c. Adjusted the mirror d. The bell curve shape is steep.
d. Reminded the class to first adjust the mirror 544. Here is a test item:
DISTANT : NEAR :: GENUINE : _______.
533. A pupil who has developed a love for reading keeps in reading for This item is a/an _________.
his enjoyment. His motivation for reading is: a. Analogy c. Metaphor
a. Insufficient c. Extrinsic b. Riddle d. Completion
b. Intrinsic d. Both intrinsic and extrinsic 545. What refers to a single word or phrase that tells the computer to do
534. Which educational trend is occurring in all modern societies as a something with program or file?
result of knowledge explosion and rapid social, technological and a. Computer language c. Command
economic changes? b. Computer program d. Password
a. Nuclear education 546. Can an insane person be blamed for killing a stranger?
b. International education a. Yes, because an insane person possesses a little
c. Lifelong learning degree of voluntariness
d. Team teaching b. Yes, because an insane person is not totally ignorant
c. No, because of his ignorance and lack of
535. With the advent of multi-media resources and computers, which is voluntariness
the most favorable result of the optimal use of educational media d. No, because the one killed is a stranger, not in any way
technology? related to him
27
547. A political boss builds a school in a distant barrio in order to get the 559. Kiko is very attached to his mother and Sharon to her father. In
votes for an unworthy and corrupt candidate. Is the action of the what developmental stage are they according to Freudian’s
political boss moral? psychological theory?
a. No, the candidate is undeserving a. Phallic stage c. Oedipal stage
b. No, his move was not meant for good effect b. Latent stage d. Anal stage
c. Yes, the votes were exchange for the school built 560. A student dislikes Math due to traumatic experience in the past.
d. Yes, it was his duty to strategies for his candidate to win Which law explains this?
548. The teacher’s first task in the selection of media in teaching is to a. Partial activity c. Vividness
determine the: b. Analogy d. Disposition/Mind set
a. Choice of the teacher
b. Availability of the media 561. Which illustrates vicarious punishment?
c. Technique to be used a. We feel so bad to a classmate who is punished for being
d. Choice of the students tardy so we convince him go to school on time
549. Which is the true foundation of the social order? b. Out of comparison, we volunteer to get punished in place
a. Strong political leadership of a friend
b. The reciprocation of rights and duties c. We charge to experience our being punished
c. Equitable distribution of wealth d. See someone get punished for habitual tardiness. In
d. Obedient citizen effect, we are less likely to be tardy
550. All the examinees obtained scores below the mean. A graphic 562. In instructional planning, it is necessary that the parts of the plan
representation of the score distribution will be: from the first to the last have:
a. Perfect normal curve a. Symmetry c. Conciseness
b. Negatively skewed b. Coherence d. Clarity
c. Positively skewed 563. If Teacher Judith has to ask more higher-order questions, she
d. Leptokurtic has to ask more ________ questions.
551. In a normal distribution curve, a T-score of 70 is: a. convergent c. fact
a. Two SDs above the mean b. closed d. divergent
b. Two SDs below the mean 564. For maximum interaction, a teacher ought to avoid ______
c. One SD below the mean questions
d. One SD above the mean a. rhetorical c. divergent
552. Which one stifles student’s initiative? b. leading d. informational
a. “Bahala na” 565. The military training requirements among students in the
b. “Utang na loob” secondary and tertiary levels can be traced as a strong
c. Rationalism influence of the:
d. Extreme authoritarianism a. Greeks c. Chinese
553. The following are trends in marking and reporting system, b. Romans d. Athenians
except: 566. Teacher Marissa wants to review and check on the lesson of the
a. Supplementing subject grades with checklist on traits previous day? Which one will be most reliable?
b. Conducting parent-teacher conferences as often as a. Having students correct each other’s work
needed b. Having students identify difficult homework problems
c. Raising the passing grade from 70 to 80 c. Explicitly reviewing the task relevant information for
d. Indicating strong points as well as those needing the day’s lesson
improvement d. Sampling the understanding of a few students
554. Which is a type of graph in which lines represent each score or 567. To promote effective practice, which guideline should you bear
set of scores? in mind? Practice should be ____.
a. Histogram a. Difficult for students to learn a lesson
b. Scatter gram b. Arranged to allow students to receive feedback
c. Scatter plot c. Done in an evaluative atmosphere
d. Frequency polygon d. Take place over a long period of time
555. Which is an example of a perfect duty? 568. Which is one role of play in the pre-school and early childhood
a. Paying the worker the wages agreed upon years?
b. Donating an amount for a noble project a. Separates reality from fantasy
c. Giving alms to the needy b. Develops the upper and lower limbs
d. Supporting a poor but deserving student to school c. Develop competitive spirit
556. In what way can instructional aides enhance learning? d. Increase imagination due to expanding knowledge and
a. Entertain student emotional range
b. Hold students in the classroom 569. Teacher Joel taught a lesson denoting ownership by means of
c. Reinforce learning possessives. He first introduced the rule, then gave examples,
d. Take the place of the teacher followed by class exercise, then back to the rule before he
557. The study on types of reading exercises gives practice in: moved it the second rule. Which presenting technique did he
a. All sorts of study methods use?
b. Reading skills needed in other project a. Sequential c. Comparative
c. Recognizing the precise meaning of words b. Combinational d. Whole-Part-Whole
d. Picking out the man ideas 570. For which may you use the direct instruction method?
a. Use a microscope properly
558. How students learn may be more important than what they b. Distinguish war from aggression
learn. From this principle, which of the following is particularly c. Appreciate Milton’s Paradise Lost
important? d. Become aware of the pollutants around us
a. Knowing how to solve a problem 571. By what name is indirect instruction or Socratic methods also
b. Solving a problem within time allotted known?
c. Getting the right answer to a word problem a. Questioning method
d. Determining the given b. Morrison method
c. Indirect method
d. Mastery learning
28
572. Which does not belong to the group of alternative learning 582. Which is the ultimate aim of classroom management?
systems? a. To set up condition that brings about effective
a. Multi-age grouping teaching and learning
b. Multi-grade grouping b. To secure conformity to rules with ease
c. Non-graded grouping c. To make children realize that they cannot do everything
d. Graded education they want
573. Student Ben was asked to report to the Guidance Office. d. To remove the physical condition in the room
Student Ben and his classmates at once remarked. “What’s 583. Under which teaching strategy does a School’s division practice
wrong?” What does this imply? of assigning a Girl Scout to serve as Superintendent of the Day
a. Reporting to a Guidance Office is often associated with or Mayor of the Day for leadership training fall?
misbehavior a. Panel discussion
b. Student Ben is a “problem” student b. Symposium
c. Guidance counselors are perceived to be “almighty and c. Simulation
omniscient” d. Dramatization
d. The parents of Students Ben must be of the delinquent 584. Which approach makes you think of your thinking?
type a. Constructivist c. Cognitive
574. A teacher combined several subject areas in order to focus on a b. Metacognitive d. Integrative
single concept for interdisciplinary teaching. Which 585. The grades make valid indicators of students’ achievements.
strategy/method did he use? Which process should be observed?
a. Unit method a. Explaining the meaning of marks and grades
b. Thematic instruction b. Defining the course objective as intended learning
c. Problem entered learning outcomes
d. Reading-writing method c. Adopting letter grades such as A, B, C and D
575. Which is a major advantage of curriculum-based assessment? d. Giving objective type of tests
a. It tends to focus on anecdotal information on student’s 586. Who are not covered by the Code of Ethics of Professional
progress Teachers?
b. It is based on a norm referenced measurement model a. All full time or part time public and private school teacher
c. It is informal in nature and administrator
d. It connects testing with teaching b. Teachers of academic, vocational, special, technical or
576. A school Division Superintendent was enthusiastically lecturing non-formal institution
on the Accreditation Program for Public Elementary Schools c. Teacher in the tertiary level
(APPES), its benefits and demands. Thinking of its many d. Teacher in all educational institutions at all levels
demands, most of the school heads were not very happy about 587. Which of the UNESCO’s four pillars of education is most related
it and the older one were whispering “we have very reason to to peace education?
retire soon.” a. Learning to do
What does this tell about the change process? b. Learning to know
a. People resist change for no reason c. Learning to live
b. People tend to resist change d. Learning to be
c. Resistance to change is insurmountable
d. Leadership can affect the desired change in persons 588. The failure of independent study with most Filipino students may
despite opposition from the persons themselves. be attributed to students’ ______.
577. In the problem solving method of teaching, which is the primary a. high degree of independence
role of the teacher? b. ambiance
a. Clarifier c. Observer c. unpreparedness for schooling
b. Judge d. Director d. high degree of dependence on authority
578. To educate the child for freedom is to educate him/her to: 589. When you use the overhead projector for topic presentation,
I. Respond to others point to the ______.
II. Respond to himself/herself a. OHP slide c. OHP screen
III. Do as he/she pleases b. OHP light d. Projection wall
IV. Live as he/she desires 590. Where do you make the correction of your notes while using the
a. II and III c. I and II overhead projector?
b. I and III d. I and IV a. On the slide
579. I want to engage my students in small group discussion. Which b. On the overhead projector
topic lends itself to a lively discussion? c. On the projector wall
a. The meaning of the law of supply and demand d. On the screen
b. Rules on subject-verb agreement
c. The law of inertia 591. Authority comes from God and is meant to:
d. The exclusion of Pluto as a planet a. Help those given the authority to do their task
580. In which of the following would programmed learning to be most b. Distinguish those with authority from those without
likely to be found? c. Be lorded over others
a. In a class divided into small groups d. Make the subjects of authority recognize their superiors
b. Independent study 592. For a discussion of a topic from various perspectives, it is best
c. In dyadic groups to hold a/an ______.
d. In a class where teacher tries to individualized a. Brainstorming c. Debate
instruction b. Symposium d. Panel discussion
581. A master teacher, the resource speaker in an in-service training,
presented the situated learning theory and encouraged her 593. A Principal tells her teacher that training in the humanities is
colleagues to apply the same in class. Which did she not most important. To which educational philosophy does he
encourage her colleagues to do? adhere?
a. Apprenticeship a. Perennialism c. Essentialism
b. Learning as it normally occurs b. Existentialism d. Progressivism
c. Authentic problem solving 594. Jonna, a principal, shares this thought with her teachers.
d. Decontextualized teaching “Subject matter should help students understand and appreciate
29
themselves as unique individuals who accept complete 606. Which Republic Act provides government assistance to students
responsibility for their thoughts, feelings and action.” From which and teachers in private education?
philosophy is this though based? a. RA 7784 c. RA 7836
a. Essentialism c. Progressivism b. RA 6728 d. RA 6675
b. Perennialism d. Existentialism
595. Who stressed the idea that students cannot learn if their basic 607. The authoritarian setting in the Filipino home is reinforced by a
needs are not first met? classroom teacher who:
a. Maslow c. Wertheimer a. Is open to suggestions
b. Miller d. Thorndike b. Encourage pupils to ask questions
596. A person, who has had painful experiences at the dentist’s c. Prescribes what pupils should do
office, may become fearful at the mere sight of the dentist’s d. Ask open ended questions
office building. Which theory can explain this?
a. Classical conditioning 608. Who among the following believes that learning requires
b. Generalization disciplined attention, regular homework, and respect for legitimate
c. Operant conditioning authority?
d. Attribution theory a. Essentialist c. Progressivist
597. One strength of an autobiography as a technique for personality b. Perennialist d. Reconstructionist
appraisal is that ________. 609. The Constitutional provision on language has the following aim,
a. It may be read by unauthorized people except:
b. It can replace data obtained from other data-gathering a. To make Filipino the sole medium of instruction
technique b. To make the regional dialect as auxiliary media of
c. It makes possible the presentation of intimate instructions in regional school
experiences c. To maintain English as a second language
d. It gives complete data about the author d. To make Filipino the national Language and medium of
598. Which Millennium Development Goal (MDG) goal is related to instruction and communication
the state’s goal for quality education? 610. The tendency to emphasize so much on school beautification to
a. 1 b. 2 c. 3 d. 4 the detriment of pupils’ performance illustrates the:
599. All of the following describe the development of children aged a. Filipino’s lack of seriousness
eleven to thirteen, except: b. Filipino’s love for “porma”
a. Sex differences in IQ become more evident c. Filipino’s lack of reflection
b. They exhibit increased objectivity in thinking d. Filipino’s sense of humor
c. They shift from impulsivity to adaptive ability
d. They show abstract thinking and judgment 611. In order to avoid disgrace, a pregnant, unmarried woman takes
600. One learns Math by building on the Math concepts previously drugs to induce abortion. Is she morally justified to do that?
learned. This is an application of: a. Yes, it can save her and child from disgrace when he
a. Constructivist c. Physiological grows up
b. Humanist d. S-R b. No, the act of inducing abortion is bad in itself
c. No, the unborn child cannot be made to suffer the
601. The singing of the National Anthem in schools is an offshoot of the consequences of the sins of his parents
philosophy of: d. No, it is better to prevent the child from coming into the
a. Nationalism c. Naturalism world who will suffer very much due to the absence of a
b. Pragmatism d. Socialism father
612. In which way does heredity affect the development of the learner?
602. The environment in order to facilitate learning must be interactive. a. By placing limits beyond which the learner cannot
Which of the following best typifies this kind of environment? develop
a. The child listens to a lecture on fossils given by the b. By providing equal potential to all
teacher c. By compensating for what environment fails to develop
b. The child goes out and discovers for himself some d. By blocking the influence of environment
rock or fossil 613. The cultivation of reflective and meditative skills in teaching is an
c. The child summarize the section on fossils in his science influence of:
textbook a. Taoism c. Confucianism
d. The child copies a list of facts concerning fossils on the b. Shintoism d. Zed Buddhism
blackboard
603. Social development means the acquisition of the ability to behave 614. A child refuse to obey orders or displays negativism as a
in accordance with: development trait. How may you best handle him?
a. Stereotyped behavior a. Detain him after office hours for him do to what he has
b. Social expectation been ordered to do
c. Social insight b. Take every opportunity to praise him for every
d. Universal norms positive attitude display
c. Insist on compliance to the same degree required of
604. When an adolescent combines ability to use deductive and pupils
inductive reasoning in constructing realistic rules that he can d. Avoid giving him orders if you do and he objects take
respect and live by, how does he perceive his environment? back the order
a. He views the world from his own perspective 615. Which term refers to the collection of students’ products and
b. He sees the world and himself through the eyes of other accomplishment for a period of evaluation purposes?
people a. Portfolio c. Anecdotal record
c. He interprets events form a limited views b. Observation report d. Diary
d. He sees events apart from himself and other people
605. Who introduced the technique of using the drawing of a man as a 616. For comparing and contrasting which graphic organizers is most
measure of intelligence? appropriate?
a. Aristotle c. Goodenough a. Cycle c. Story map
b. Herbert d. Binet b. Web d. Venn Diagram

30
617. Laging UMUUKILKIL sa isipan ng ama ang nasirang pangako ng d. Ask pupils to submit test questions or reactions which
anak. you can select topics
a. Sumasagi c. Bumubuhay 627. Which is not a characteristic of a democratic discipline?
b. Gumugulo d. Sumasapi a. Child has opportunity to expense his/her opinion
618. Huwag kang maniniwala sa bulaklak ng kanyang matamis na dila: b. Child’s given punishment is related to the misdeed
a. Ito’y panunukso c. Child understands the meaning of rules
b. Ito’y pambobola d. Child obeys blindly
c. Ito’y pagbibiro 628. Who among the following stressed the processes of experience
d. Ito’y pagsisinungaling and problem solving?
a. Dewey c. Hegel
619. If a resilient child with superior intelligence is reared in a poor b. Aristotle d. Plato
environment the probable outcome would be: 629. Which of the following reasons of measuring student achievement
a. No change in IQ because environment deprivation has is not valid?
nothing to do with intelligence a. To prepare feedback on the effectiveness of the learning
b. Mental retardation since he is culturally deprived process
c. Slight change in IQ although he can overcome b. To certify that students have attained a level of
frustration and obstacle competence in a subject area
d. Great change in IQ because he is culturally deprived c. To discourage students from cheating during test
620. Which of the following is usually considered the most important and getting high scores
factor in a child’s observable classroom behavior? d. To motivate students to learn and master the materials
a. Intelligence c. Self concept they think will be covered by the achievement test
b. Heredity d. Cultural background 630. Which characterizes the perfectionist type of students?
a. Does not volunteer or initiate
621. Section 5, Article XIV, of the Constitution states that academic b. Give up easily
freedom shall be enjoyed in: c. Rarely complete tasks
a. Public assemblies d. Often anxious, fearful or frustrated about quality of
b. All institution of higher learning work
c. State colleges and universities 631. When a school decides to work on a thematic curriculum which
d. All levels of learning should be out of the picture?
a. Peer collaboration
622. A teacher who subscribes to the pragmatic philosophy of b. Integration
education believes that experience should follow learning in her c. Team teaching
teaching, she therefore exerts effort in: d. Competition
a. Encouraging learners to memorize factual knowledge 632. Teacher Mary wants to teach her pupils the technique on reading
b. Providing learners opportunities to apply theories for information. Which technique should be used?
and principles a. Text structure c. Story map
c. Equipping learners with the basic abilities and skills b. Prior knowledge d. SQ3R
d. Requiring learners full mastery of the lesson 633. In instructional planning, which among these three: unit plan,
course plan, lesson plan is (are) most specific? _________ plans.
623. As a parent and at the same time a teacher, which of the following a. Course and lesson c. Lesson
will you do to show your cooperation to a PTA project in your b. Course d. Unit
school to be financed with the proceeds of the sales of the school 634. The use of drills in the classroom is rooted on Thorndike’s law of:
canteen where food prices are little bit higher? a. Readiness c. Effect
a. Bring food for you and your children, but always b. Exercise d. Belongingness
make it a point to buy in the school canteen 635. Positive interdependence as an element of collaborative learning
b. Buy all your food in the school canteen but request for a means that the students must:
discount a. Learn to depend on each other to achieve a goal
c. Bring food enough for you and your children but do not b. Depend on the diligent students
eat in the canteen c. Help one another in the individual test for everyone to
d. Buy all your food from the school canteen even if you pass
cannot afford to do every day d. Be grouped heterogeneously
624. How can you help a habitual borrower of money get rid of his
habit? 636. Which computer seems to have the most potential for the
a. Let him do something for you in return for the money you classroom?
lent him a. Mainframe computer
b. Direct him to others b. Minicomputer
c. Do not lend him anymore c. Microcomputer
d. Ask for a collateral for the cash he is loaning d. LPC
625. Periodic checks on student seatwork with a smile and pat on the
shoulder effectively reinforce good study habit is an example of: 637. A teacher notices glaring wrong pronunciation of vowel sounds
a. Discrimination reinforcement among her students necessitating more practice. Which of the
b. Variable-ratio schedule following activities would be most helpful?
c. Continuous reinforcement a. Dictionary use c. Assignments
d. Fixed interval and variable-interval schedule b. Review d. Drill
626. A person strives to work at a given task because of a need. Which
of the following situations can make a person strive to meet his 638. What storage device is significantly more efficient in holding
needs? information?
a. Minimize the unpleasant consequences of student a. Hard disk c. Floppy disk
involvement b. Software d. Audio cassette
b. Utilize your own opinion as teacher in making final 639. The Filipino tendency to resort to the easy way out from a term
decisions in the classroom paper as a course requirement by hiring a ghost writer or by
c. Use unfamiliar materials as examples in order to initially passing a photocopied term paper provide which Filipino traits?
arouse their curiosity a. Anticipation c. Pakikisama
31
b. Ambivalence d. Lack of discipline 650. Pick out the situation that illustrates the duty of a new teacher to
640. “No pain, no gain.” This means that: the state:
a. One should be penitent every Friday by carrying his a. Take a long vacation which she firmly believes she
cross deserves after four years of diligent study before taking
b. Only those willing to carry the crosses imposed can the examination for teachers
share the joy of life b. Take the licensure examination for teacher and an
c. The more suffering in this life, the more one is assured of oath to do her best to help carry out the policies of
heaven the state
d. One should look for suffering to save himself/herself c. Apply for teaching job where eligibility is not required to
641. Which trust on value formation is meant to help the students make gain teaching experience before taking the teachers
use of their thinking and scientific investigation to decide on topics board examination
and questions above values? d. Prepare for the wedding she and her boyfriend have long
a. Value inculcation c. Value clarification planned to able to raise a family with children which they
b. Analysis d. Moral development plan to rear as good citizen of our country
651. Parents are up in arms on the telephone bills that pay for sex calls.
642. Which interactive teaching should be avoided? What is the solution to this problem?
a. Using multiple response strategy a. The telephone company is to blame for this
b. Using “put down” strategy b. Parents, school and students should discuss this
c. Asking more divergent questions openly
d. Asking more evaluative questions c. The government restriction have no teeth
d. Parents allow this to make their children modern
643. Rights which cannot be renounced or transferred because they are
necessary for the fulfillment of man’s primordial obligations are 652. Cooperative is encouraged in as many groups as possible. What
called: agency controls the different cooperatives?
a. Alienable rights c. Inalienable rights a. Security and Exchange Commission
b. Perfect rights d. Acquired rights b. Department of Local Government
644. Which is in line with equitable access to education but runs c. Commission on Audit
counter to quality? d. Bureau of Cooperative
a. Selective retention of students
b. Deregulated tuition fee hike 653. Society and media know drinking starts off drug addiction. What
c. Open admission should be discussed in schools?
d. Program accreditation a. Nobody drinks at home except father
645. A negative discrimination index means that: b. Drug addiction has been traced to drinking wine
a. The items could not discriminate between the lower and c. TV ads show drinking is a source of fellowship
upper group d. High taxes on liquor will be deterrent to eventual drug
b. More from the lower group answered the test items use
correctly 654. Cooperatives have branched out to consumers cooperative.
c. More from the upper group answered the test item Schools have included the concepts of cooperatives. Where is it
correctly practiced?
d. Less from the lower group got the test item correctly a. School book stores
b. Schools uniform purchases
646. Your teacher is of the opinion that the world and everything in it c. School canteen
are ever changing and so teaches you the skill to cope with the d. Class stores
changes. What is his governing philosophy?
a. Experimentation c. Realism 655. A student collapsed in her social studies class. It was found out
b. Existentialism d. Idealism that he did not eat her lunch. What principle is shown in the
situation?
647. For brainstorming to be effective which one should be out? a. Psychological need
a. Making use of the others ideas shared b. Physiological need
b. Teacher’s judge mental attitude c. Psychosomatic
c. Non-threatening atmosphere d. Safety need
d. Openness to idea 656. The main function of a philosophy of education is to:
a. Aid the learner to build his own personal philosophy
648. Which statement on spaced and massed learning is correct? b. Reconsider existing educational goals in the light of
a. Massed learning is better than spaced learning society’s needs
b. Massed learning is as effective as spaced learning c. Provide the academic background prerequisite to
c. Spaced learning is better than massed learning learning
d. Both massed learning and spaced learning are not d. Define the goals and set the direction for which
effective education is to strive
657. Which technique/s enable/s a teacher to identify and eventually
649. Which of the following measures should a teacher do to a principal assists students with interpersonal difficulties?
whom she would like to file a case of sexual harassment without a. Anecdotal record c. Cumulative record
violating the relationship of the teacher and her superiors? b. Personal inventory d. Sociogram
a. Write an anonymous letter to a higher school official to 658. Teachers and students can participate in levels of computer use.
denounce the superior Give the order of computer use from simplest to complex?
b. Present the case before a competent authority and a. Computer competency, computer literacy, competency
prepare to prove the charge expertise
c. Call a parent-teacher meeting and denounce the b. Computer literacy, computer competency, computer
superior expertise
d. Encourage the other teachers and students to hold a c. Computer literacy, computer expertise, computer
demonstration to oust the superior competency
d. Computer competency, computer expertise, computer
literacy
32
659. Which one is considered the “Brain” of the microcomputer? d. Keeps himself abreast with educational trends
a. CPU c. Video Screen 670. Which one indicates a teacher’s genuine enthusiasm and pride in
b. Software d. Keyboard teaching?
a. Sticking to teaching for the moment that there are no
660. A group activity wherein one group representative presents the better offers
output to the bigger group rather than individual pupils presenting b. Telling everyone that he went to teaching for there was
the output is known as: no other choice then
a. Consensus decision c. Jury trial c. Engaging himself in continuing professional
b. Composite report d. Agenda education
d. Belittling the remuneration one gets from teaching
661. In the formulation of classroom regulations, which of the following 671. In writing performance objective which word is not acceptable?
should a teacher refrain from doing? a. Manipulate c. Delineate
a. State classroom regulation as clearly as possible b. Integrate d. Comprehend
b. Teacher and the class should make as many 672. When is giving praise ineffective? When it?
regulations as possible a. Uses the accomplishment of peers as the context for
c. Enlist student aid in the formation of classroom describing a student’s present accomplishment
regulation b. Provides information to student’s about their competence
d. Enforce classroom regulations consistently and fairly and the value of their accomplishment
c. Focuses students attention on her own task relevant
662. Zero standard deviation means that: behavior
a. The students scores are the same d. Shows spontaneity, variety and other signs of credibility
b. 50% of the scores obtained is zero 673. Which statement applies when scores distribution is negatively
c. More than 50% of the score obtained is zero skewed?
d. Less than 50% of the scores obtained is zero a. The mode corresponds to a lower value
b. The median is higher that the mode
663. Which is the least authentic mode of assessment? c. The mode and median are equal
a. Paper-and-pencil test in vocabulary d. The mean corresponds to a high value
b. Oral performance to assess student’s spoken 674. The use of the process approach gives the student the opportunity
communication skills to:
c. Experiments in science to assess skill in the use of a. Learn to their own
scientific methods b. Apply the scientific method
d. Artist production for music or art subject c. Make use of laboratory apparatuses
d. Learn how to learn
664. In what period of a child is physical growth fastest? 675. A comprehension skill of higher level which may be inferred or
a. Prenatal period implied from reading is:
b. Early adolescence a. Picking out the main idea
c. Early childhood b. Drawing conclusion
d. Prenatal and early adolescence c. Nothing specific details
d. Following direction
665. How does fear affect the voluntariness of an act? 676. In the Preamble of the Code of Ethics of Professional Teachers,
a. Makes the act involuntary which is not mention about teachers?
b. No effect at all a. Dully licensed professionals
c. Increases voluntariness b. Posses dignity and reputation
d. Lessens but not destroy voluntariness c. LET passers
d. With high moral values
666. A group of people asserts that their culture is superior to another. 677. What does a skewed score distribution mean?
This exemplifies: a. The scores are concentrated more at one end or the
a. Cultural gap other end
b. Ethnocentrism b. The mode, the mean and the median are equal
c. Cultural conflict c. The mean and median are equal
d. Norm conflict d. The scores are normally distributed

667. A test consists of a graph showing the relationship between age 678. What is implied by a negatively skewed score distribution?
and population. Follow a series of true-false items based on the a. The scores are evenly distributed from the left to the
graph. Which type of test does this illustrate? right
a. Laboratory exercise b. Most pupils are underachieves
b. Interpretative c. Most of the scores are high
c. Problem solving d. Most of the scores are low
d. Performance
668. Which curricular move served to strengthen spiritual and ethical 679. A teacher discovers that a product of a certain bottling company
values? brings about damage to teeth. Much as he wants to share the
a. Integration of creative thinking in all subject products of his research, he could not because of harassment from
b. Introduction of Value Education as a separate all sides. Which teacher’s right is violated?
subject area a. Right to property
c. Reducing the number of subject areas into the skill b. Academic freedom
subject c. Right to one’s honor
d. Re-introducing Science as all subject in Grade 1 d. Right to make a livelihood
669. A teacher is a facilitator of learning and of the development of the 680. Why can the calculator do arithmetic? Because:
youth. Which practice is not keeping with his role as facilitator? a. A computer inside the calculator tells it how
a. Considers the multiple intelligences of learners b. A watch inside direct it
b. Humiliates misbehaving pupils c. A typewriter inside does it
c. Dialogs with parents and with other members of the d. A TV inside shows it
community
33
681. On which constitutional provision is the full or partial integration of b. On line research
capable deaf and blind students in the classroom based? The c. Manual research
provision on: d. Computer research
a. Protecting and promoting the right of all citizen to
qualify education 692. What best indicates the effectiveness of classroom activities?
b. Providing citizenship and vocational training to adult a. The laughter and enjoyment of students
citizen b. The application of concept learned in daily life
c. Academic freedom c. The utilization of varied techniques and approaches
d. Creating scholarship for poor and deserving students d. The variety of instructional materials used
682. Teaching in the cognitive, psychomotor and effective domains is
based on the concept that the learner is a: 693. The main purpose of the compulsory study of the Constitution in
a. Moral and feeling being Philippine schools is to:
b. Maternal and an acting being a. Develop the students into responsible thinking
c. Thinking, feeling and acting being citizens
d. Spiritual and maternal being b. Acquaint students with the historical development of the
683. Both Muslim and Christian value marriage but the Muslim practices Philippine Constitution
polygamous marriage while the Christian practices monogamous c. Prepare students for law making
marriage. What is this called? d. Make constitutional experts of the students
a. Cultural relativism c. Ethical relativism 694. Some students who are high in the scholastic aptitude test have
b. Acculturation d. Enculturation failed in college. Some who are below the standards set for
684. Teacher wants to compare 2 concepts. With which technique can admission but who for various reasons were admitted, attained
accomplish this best? satisfactory standings. This proves that:
a. K-W-L technique c. Spider web a. Human beings are certainly predictable
b. Venn diagram d. Histogram b. Admission tests are not accurate, hence should not be
685. To build a sense of pride among Filipino youth what should be used
done? c. Aptitude tests do not measure all factors important
a. Re-study our history and stress on our for success
achievements as a people d. Aptitude test can be perfectly relied on
b. Set aside the study of local history 695. If the teachers pattern in questioning consists of calling on a
c. Re-study our history from the perspective of our student then asking the question:
colonizers a. All students may be encouraged to participate
d. Replace the study of folklores and myths with technical b. The student called to answer may be able to think well of
subjects his answer
686. When necessary conditions are present, the use of inductive c. The rest of the class may just dictate the answer
method is preferred because: d. The rest of the class may not engage themselves in
a. It gives the teacher more time to rest thinking of the answer
b. There is greater active participation on the part of 696. Freud expounded that there is a period when young girls
the pupils experience rivalry with their mother for their father’s affection. This
c. It needs only few instruction materials is called:
d. Academic time is used wisely a. Electra complex
b. Oedipus complex
687. Which is the best reason why teacher begins a lesson in Math by c. Achilles syndrome
checking and reviewing on the previous day’s assignment and d. Cassandra syndrome
provides practice and drills? 697. Education is a lifelong process. This simply means that education:
a. Check if parents guide their children in the making of a. May take place formally or informally to enable the
assignment individual to grow
b. Make sure that the students understand the pre- b. May take place anywhere and anytime the individual so
requisite skills of the lesson desires
c. Prepare the students for the mastery test c. Is a continuous process of experiencing and
d. Make learning interesting and enjoyable for students reorganizing experiences
d. Take place in the school where the individual is exposed,
688. Which is a selective reading technique meant at getting at self contained experiences
important facts very fast? 698. The tendency to imitate elders is very strong in the early childhood
a. Skim reading c. Oral reading stage. Teachers should therefore be very good:
b. Scanning d. Silent reading a. Counselors c. Disciplinarians
689. For counseling to be successful which assumption must be b. Role models d. Facilitators of learning
avoided? 699. How is Values Education offered in the National Secondary
a. The environment must provide assurance of Education Curriculum?
confidentiality a. Emphasized in Science and Technology
b. The counselor tells the student what to do b. Integrated in all subject areas
c. The student is willing to participate in the process c. As a separate subject
d. The counselor must be able to relate to the student d. Integrated with Technology and Home Economics
700. The NSEC orients secondary education to:
690. Which technique is most appropriate when a teacher wants a a. The teaching of the national symbols
group to agree on a plan of action? b. Health values development
a. Composite report c. The development of competencies and values for
b. Consensus decision making social living
c. Symposium d. National development requirement and reflects search
d. Agenda based direction
701. The child cannot distinguish abstracts during the sensory motor
691. What term applies to the search for related literature by computing of development. Which of these techniques should a teacher
access of databases of discs kept in libraries? apply to accommodate learning?
a. Compact discs computer research a. Make use of individualize instruction
34
b. Explain the lesson very well b. Multiple choice d. Essay
c. Utilize concrete objects to clarify concept 713. In a multiple choice test, keeping the options brief indicates
d. Provide variety of educational toys ____________.
702. Which of these systems of learning includes ways and methods a. Inclusion in the item irrelevant clues such as the use in the
which are used in preserving and building certain within cultural correct answer
communities? b. Non inclusion of option that mean the same
a. Non-formal learning c. Plausibility and attractiveness of the item
b. Multi-level learning d. Inclusion in the item any word that must otherwise
c. Cultural learning repeated in each response
d. Indigenous learning 714. Which of these criteria is the most important in test
constructions?
703. Which of the following statement is true in the use of a. The stem should contain the central problem
experiments and demonstrations in teaching Science: b. Items should be congruent with the objectives
a. It is valuable if used in the context of a lesson that c. A table of specification should be prepared
related observation to other information d. Options should be of almost the same length
b. It should be encouraged in elementary school since the 715. Which of these philosophers is reflective of that of Dewey’s
concept the encompass are difficult for your children which stresses the development of an individual capable of
c. It is as valuable as teaching by lecturing reflective thinking specifically that of being able to solve the
d. It is less valuable than teaching through inquiry and problem be faces individually or collectively?
discussion a. Disciplinarianism c. Experimentation
b. Developmentalism d. Rationalism
704. Identical twins are more alike than fraternal twins. Which of the 716. Which of the following abilities is stressed by humanistic
following statement principle supported by this? education?
a. Environment affects both fraternal and identical twins a. Learn the different philosophies of education
b. Intelligence hinges in physical structure b. Develop man into a thinking individual
c. Heredity has a part in determining intelligence c. Enjoy the great works of man such as the classics
d. Intelligence is determined partly by pre-natal nutrition d. Make man distinctly civilized, educated and refined
717. An appreciation lesson is one that is designed to lead the class
705. In testing which of the following is referred to as cultural bias? to conduct and enjoy something. Which of the following
a. Some culture do better on test than others statements closely approximate the meaning of the above?
b. Test items are more familiar with some culture a. An appreciation lesson should be a lesson in values
c. Test will show who is more cultured b. Appreciation lessons help pupils weigh and clarify values
d. Cultured people do better on tests c. One cannot fully appreciate what one does not
706. Which is the most obvious and familiar way of reporting understand or enjoy
variability? d. A teacher should plan lessons that will guide children
a. Standard deviation to appreciate what is beautiful
b. Range between highest with some culture 718. Which of the following is the best time for a teacher to set up
c. Standard error of the mean routine activities that will contribute to effective classroom
d. Distribution of raw scores management?
707. The theory of identical elements in learning holds that transfer is a. As soon as the students have established
facilitated when the: b. Daily at the start of the session
a. Teacher uses different teaching devices c. During his homeroom days
b. Learner has a memory of specific responses d. On the every first day of school
c. Development task is easily identified
d. Experience is similar to the application situation 719. In large classes where little of the work pupils can be
individualized, the most effective and practical ways to
708. If this need is not met, the adolescent tends to be critical and individualize instruction is to:
always tries to find fault. This is the need: a. Devise group activities which afford every pupils an
a. For adventure opportunity to work at his own
b. For recognition b. Give the pupils freedom to launch individual projects
c. To belong c. Assign homework and check it regularly
d. For material security d. Assigned program material for out-of-class hours
709. The way a child talks and walks manifest gestures that have
been learned from models he had been exposed. This explains 720. Which of these is the most important principle that a teacher
what influence? should follow in initiating a program with positive reinforcement?
a. Affective c. Social a. Make sure the reward comes immediately after the
b. Insight d. Cognitive appropriate behavior
b. Punish negative behavior and reward positive behavior
710. Audio-visual aids are used in classroom teaching to c. Provide regular opportunity for socially acceptable
__________. behavior
a. Help make learning more permanent d. Consider peer approval and recognition
b. All of these
c. Help clarify important concept 721. The trend of focusing attention on the child’s interests, abilities
d. Arouse and sustain student’s interest and needs and on the improvement of community living
711. Which of the following is the most important purpose for using necessitate the use of the:
achievement test? To measure the _________. a. Discovery approach
a. Quality and quantity of previous learning b. Conceptual technique
b. Quality and quantity of previous teaching c. Integrative approach
c. Educational and vocational aptitude d. Project method
d. Capacity for future learning
712. Which of the different types of test covers a wide variety of 722. The best way the teacher can be of the appropriateness of an
objectives? instructional materials is to:
a. True-false c. Matching a. Try it out before using it in class
35
b. Consider its technical quality 731. When do test, inventories and career information become
c. Consider its availability effective for counseling services?
d. Consider its cost a. When the data generated are interpreted on time by
professionally competent person
723. Tasks analysis involves the breaking down of a learning task b. The psychological test result are still valid and reliable
into subtasks or sub skills. Given a task to retell a story, which of c. When the records are updated
the following skills is not needed? d. When the records are kept for ready reference when
a. To disseminate information needed
b. To outline a selection 732. Which of the following is a major advantage in using arithmetic
c. To identify topic sentences mean?
d. To arranged events in sequence a. It is more commonly used than other measures
b. It is simple to compute
724. You are assigned to teach students with varied abilities. You c. It discriminates between the lowest and the lowest
want to teach a more homogenous grouping. Which type of d. It is more than stable than the median
grouping will tend to benefit your students? 733. When I am engaged in an external criticism in a historical
a. Mixed ability grouping research, what am I occupied with? The _______ of the
b. Low ability group document.
c. Within class ability grouping a. Author c. Source
d. High ability grouping b. Authenticity d. Accuracy
734. Learners often find it much easier to fit into a new social
725. Which of the following examples illustrate the use of questions situation when given encouragement and support. How can this
to focus pupils attention on the key points of the lesson? be done?
a. What is Rizal Park known for? a. By discovering his new interest
b. Why are machine made goods cheaper than those b. By giving him room responsibility
made by hand? c. By assigning “peers or Buddies” to him
c. Have you ever enjoyed watching the clouds on a bright d. By giving him special help
day? 735. Who expounded on the need to study the child carefully for
d. Who came while I was writing on the blackboard? individualized instruction?
a. Da Feltre c. Boccacio
726. The new teacher entered a noisy classroom. She shouted b. Erasmus d. Ascham
immediately at the students desperately trying to get order and 736. Which of the following should a teacher do if she cannot pay the
discipline. Since then the teacher has not controlled the class. monthly installment of an appliance she got from a department
Which is the most probable cause of the teacher’s failure? store in their town?
a. The students reaction to the teacher is the consequence of a. Reject any notice of demand for payment to make the
her behavior impression that she did not receive
b. Rules are not defined and procedures to sustain order is b. Move to another neighborhood to escape payment
not put into place c. Inform the manager of the store personally and make a
c. The new teacher wants to show the class who is authority satisfactory arrangement of payment on or before the
d. The class wants to test the ability and patience of the due date of payment
teacher d. Offer the return of the used appliance to the store on the
condition that she will be refunded on the monthly
727. The educational implementation of research findings relative to installment she paid
the ability of dull learners and bright learners to organize and 737. Which of the following will you recommended to a senior high
generalize is for teacher: school scholar who is impregnated by a fellow student?
a. To make the bright learners guide the dull ones in learning a. Tell her parent about her condition
to generalize b. Stop schooling until after she gives birth
b. To make the bright learners to generalize and the dull ones c. Direct her to an abortion clinic
to memorize d. Force her boyfriend to marry her
c. To give the dull learners to more concrete experiences to 738. The government prescribes a higher percentage on the
serve as basis for generalizing administration of educational institution to Filipino citizens in
d. To give both the dull and bright learners concrete and order to:
abstract experiences to serve as basis for generalizing a. Minimize the unemployment problem
b. Produce globally competitive graduates
728. Which of the following will do the first to establish good class c. Protect the rights of the citizen
management? d. Ensure the teaching of Filipino
a. Discuss the required rules for proper class behavior 739. Teacher should bear in mind that the period of greatest mental
b. Discuss the work plan for the year development is from:
c. Prepare a seat plan a. 9 to 12 years c. 6 to 9 years
d. Train the class in the distribution of material b. 12 to 15 years d. 3 to 6 years
740. Which of the following is the best situation wherein you can
729. A student was diagnosed to have a high IQ but is failing in his balance responsibility and accountability?
academic subject. What should the teacher do to help him? a. A teacher paid on an hour basis, takes her time with the
a. Talk to his parents subject matter until the end of the period
b. Examine his study habits b. A teacher paid on an hour basis, teaches as much as
c. Talk the student and find out his problem she could for the duration of the period
d. Refer him to the guidance counselor c. A teacher paid on an hour basis, spends most of the time
on the latest gossips in showbiz
730. Which is the true foundation of the social order? d. A teacher paid on an hour basis, entertain her students
a. Strong, political leadership with stories until the end of the period
b. The reciprocation of rights and duties 741. You have a pupil who is so talkative, naughty and aggressive
c. Equitable distribution of wealth that he is a burden to the entire members of the class. How
d. Obedient citizenry would you remedy this problem?
a. Talk to him seriously
36
b. Call the parents for dialogue 755. Student David was asked to report to the guidance office. Student
c. Report the case to the principal David and his classmates at once remark: “What’s wrong?” what
d. Reprimand him always does this mean?
742. What should a teacher do before constructing items for a a. Guidance counselor are perceived to be “almighty and
particular test? omniscient”
a. Prepare a table specifications b. The parents of student Jay must be of the delinquent type
b. Review the previous lessons c. Reporting to a Guidance Office is often associated with
c. Determine the length of time for answering it misbehavior
d. Announce to students the scope of the test d. Student Jay is a “problem” student
743. Under which of the multiple choice type of test can this question 756. Which of the following assessment techniques best assess the
be classified? “Which of the following statement expresses this objective “ plans and designs an experiment to be performed”
concept in different forms?” a. Rating scale c. Checklist
a. Association c. Difference b. Paper and pencil test d. Essay
b. Definition d. Cause 757. What type of measure of variation easily affected by the extreme
744. Of the following types of test which is the most subjective in scores?
scoring? a. Quartile deviation
a. Matching type c. Multiple choice b. Standard deviation
b. Simple recall d. Essay c. Range
745. In which of these research methods can the researcher control d. Inter quartile range
certain variable? 758. A child who is cold towards that people among him might have
a. Experimental c. Descriptive failed to attain what basic goal based on Erickson’s theory on
b. Ex post facto d. Historical psychological development?
746. During the first grading period, a student obtained failing marks a. Autonomy c. Initiative
in five academic subjects. Which of the following tests would b. Trust d. Mistrust
best explain his performance? 759. Under the learning to do, which of the following instruments must
a. Mental ability c. Attitude be acquired so that a person can perform his work effectively?
b. Personality d. Aptitude a. Competence c. Compromise
747. Measuring the work done by gravitational force is a learning b. Insights d. Communication
task. At what level of cognition is it? 760. What do you think would be the actions of a teacher who found out
a. Application c. Evaluation and has proven that his principal is involved in the malversation of
b. Knowledge d. Comprehension funds of their school?
748. Setting up criteria for scoring test is meant to increase their a. Malign him trough an anonymous letter
________. b. Present the charge to a complete authority
a. Objectively c. Validity c. Ignore what the teacher has discovered about this action of
b. Reliability d. Usability the principal
749. Which of the following you will do to an examinee you caught d. Circulate this issue and let it become a gossip
cheating and who offered a certain sum of money to keep quiet? 761. Standard deviation is to measure of variations as ______ is to
a. Confiscate his test paper and report him to the measure of central tendency.
examination supervisor a. Quartile deviations c. Mean deviation
b. Motion him to keep quiet and watch for him after the b. Range d. Mode
examination 762. Which statement is/are true in constructing matching type of test?
c. Announce to all examinees the name of the cheater I. The option and descriptions not necessarily homogenous
d. Ignore him but let him feel you saw him II. Description in Column A and options in Column B
750. Which of the following is the best situation wherein you can III.The options must be greater than the description
balance rights and authority?
IV. The directions must state the basis of matching
a. Allow all their only daughter’s suitor to come and go as she
pleases a. I, II, and III c. I, II, and IV
b. Caution their only daughter’s choice of a boyfriend b. II, III, and IV d. I, II, III, and IV
c. Censor all their only daughter’s suitor 763. Which of the following should be AVOIDED in constructing true or
d. Choose a life-partner for their only daughter false test?
751. Classical conditioning theory is always attributed to him for his I. Verbal clues and specific determiner
experiment involving the dog’s salivation as a reaction to the
sound of the buzzer. II. Terms denoting definite degree of amount
a. Skinner c. Bandura
b. Lewin d. Pavlov III. Taking elements directly from the book
752. The singing of national anthem is an offshoot of the philosophical
ideas of: IV. Keep true and false statement the same in length
a. Naturalism c. Socialism
b. Nationalism d. Pragmatism a. I and III only c. I, II, and IV
753. An adolescent combines his ability to use deductive and inductive b. I, II and III d. II and IV only
reasoning in realistic rules that he can respect and live by. When 764. The discriminating index number 15 is 0.44 this means that:
he does this, how does he perceive his environment? a. Equal number of student got the correct answer
a. He sees the world through the eyes of the people b. More students from the upper group got the item
b. He interprets events from a limited point of view correctly
c. He sees events apart for himself and other people c. More students from the lower group got the item correctly
d. He views the world from his own perspective d. The test item is very easy
754. Which of the following statement is the main reason why should 765. Some children go through a period of intense appetite when they
negative words be avoided in constructing multiple choices tests? eat or chew on all sorts of inedible substances. This is called:
a. Might be overlooked a. Pica c. Encopresis
b. Stems tends to be longer b. Enuresis d. Anorexia
c. More difficult to construct option 766. What is the main advantage of using table of specification when
d. Increase the difficulty of the test item constructing periodic test?
a. It reduces the scoring time
37
b. It improves the sampling of content areas 775. The freedom constitution which provided the present philosophy of
c. It makes test construction earlier education was the contribution of the Aquino administration. Which
d. It increase the reliability of the test result of this statement is NOT consistent with our education philosophy?
767. If a teacher is an advocate of banking concept in Education he or a. Restore Ethical and moral values
she viewed student as? b. Appreciate the roles of foreigners on the historical saga
a. Clear account to be filled up by the teacher of the country
b. Dormant account to be activated by the teacher c. Foster nationalism and patriotism
c. Wobble account to be balanced by the teacher d. Introduce vocational and scientific efficiency
d. All of the above 776. In the preamble of the Code of Ethics for Professional Teachers,
768. Teacher Maechelle is a neophyte teacher. One time a mother of which of the following teacher descriptions is included in the Code
one of her students confronted and maligned her in front of her of Ethics of the Professional Teachers?
colleagues. How should conduct Teacher Myla react on this kind of a. With satisfactory teaching performance
situation? b. Duly licensed professional
a. Walk away and ignore the mother c. Persons of dignity and reputation
b. Answer back the mother and malign her too d. Passed the Licensure Exams for teacher
c. Wait until the emotion of the mother subsides and invite 777. A number of researchers found the effects of maternal
her to discuss the concern with the principal or guidance employment on children’s achievement are:
counselor a. Fully establish c. Positive and negative
d. Allow the mother to keep on maligning her until it’s her turn to b. Negative d. Hardly establish
do the same 778. Babyhood is often referred to as a “critical period” in the
769. Ms. Teodora is always guarded by the principle that she has a development of personality because:
foremost responsibility as a teacher. Given the following which do a. Changes in the personality pattern take place
you think is the main responsibility of Ms. Sanchez? b. At this time the foundations are laid upon which the adult
a. Ask the challenging question personality structure we built
b. Guides students in learning process c. The brain grows and develops at such an accelerated rate
c. Shares responsibility in counseling during babyhood
d. Inspires student to interesting lessons d. At the time the baby is exposed to many hazards both
770. Which of the following would best describe the role of the schools? physical and psychological
a. To educate the citizens 779. Research established that complete coordination of motor
b. To educate the citizens to change the society activities is attained at:
c. To fit the citizens into the society a. Childhood stage c. Pre-natal stage
d. All of the above b. Infanc d. Adolescence stage
771. Manual aesthetic activities involving attitudes and feelings are 780. Which of these theories holds that human activity is based on the
primarily expressive of emotions and values not thoughts. An interaction of stimuli and responses?
example of this motor skills is: a. Vector c. Association
a. Baking a cake with background music b. Social learning d. Cognitive field
b. Saving a board for the wall of a book cabinet 781. It is the process by which an organism inherent the characteristics
c. Dancing and playing musical instruments traits of the patients:
d. Manipulating a bowling ball to achieve a strike a. Fertilization c. Maturation
772. Parents and teachers are considered as authorities and models by b. Heredity d. Development
children at the early childhood stage. What does this statement 782. When the daughter is completing with the father for the fathers
imply? attention, the daughter is said to be experiencing:
a. Parent Teacher conference should always be an activity in a. Sexual deviation c. Identity crisis
school b. Electra complex d. Oedipus complex
b. Parents should enforce strict discipline at home and teachers 783. If children are cooperatively engaged with the teacher in a group
in school project the children will discipline themselves as each member of
c. Teachers and Parents should serve as role models at all the group exercises:
times a. Special interest
d. Teachers should demand complete obedience from the b. Moral compulsion
learners in school c. Obedience to the teacher
773. Which of these statements regarding professional’s teachers is the d. Peer influence
major difference in the professionalization of teachers and 784. Operation “Return to the basic” was launched by the department of
teaching as promulgated in Presidential Decree 1006 and in education, Culture and sports not only to upgrade pupil
Republic Act 7836? achievement but also to:
a. Assigned at the tertiary level in both private and state a. Emphasize the importance of the 3Rs
colleges and universities b. Develop thinking skills
b. Assigned at the elementary and secondary levels in both c. Encourage pupils to study
public and private schools d. Serve as the basis for learning continuum
c. Holder of valid professional license and certificate of 785. Which of the following embodies the operation “return to the
registration Basics”?
d. Appointed on full-time basis and on permanent status a. National Secondary Achievement Test
774. Social stratification is greatly developed in the classroom. Which of b. New Elementary School Curriculum
these activities would be an effective way of avoiding or minimizing c. New Secondary Education Curriculum
this? d. National Elementary Achievement Test
a. Encourage higher education aspirations among the less 786. In a classroom it is possible to see the teacher doing the following
privileged pupils to faci9litate learning.
b. Provide limited experience to children of less privileged I. The class reads a workbook on the characteristics of animals
classes II. The class copies the characteristics of animals from books
c. Assign leadership roles to the children of the upper social III. The class goes out to the zoo to observe the animals
class
IV. The teacher shows posters of animals
d. Avail the influence of mass media for children of all classes
Which of these teacher’s activities reflects an interactive
environment?
38
a. III and IV c. I and IV c. Ask a third party to write the anonymous letter to prevent her
b. I and II d. III only from being involved
787. As provider for the education act of 1982, how much are the d. Talk to the married man with whom Ms. Del Mundo is having
institutions of learning encouraged to set higher standards of an illicit affair
quality over and above the minimum required for state recognition? 798. The principal ask his good teacher to write modular lesson in
a. Formal education Filipino, then he had them published with his name printed as
b. Academic freedom author. Which is unethical in this case?
c. Voluntary accreditation a. He burdened the teachers with work not related to teaching
d. Continuing Professional Education (CPE) b. He got the merit which was due for his teacher-writer
788. The following is TRUE in the development of understanding in c. He had the modular lessons published when they worth
early childhood except: publishing
a. Sensory experiences d. He wants to exclusive beneficiary of the royalty from the
b. Abilities to reason & to see relationship modules
c. Ability to ask questions 799. Teacher Vincent, a teacher for thirty two years, refuses to attend
d. Ability to explore their environment seminars. He claims that his thirty two years of teaching is more
789. The first kindergarten also known as “a garden where children than all the seminars he is asked to attend. Are his actuation and
could grow” was the product of research by: thinking in accordance with the code of Ethics for professional
a. Froebel c. Pestalozzi teachers?
b. Herbart d. Rousseau a. Yes, provided he has an excellent performance rating
790. Values development is integrated in all subjects in the NSEC while b. No, non attendance to seminars would mean no increase in
values education is: salary
a. Emphasized in science and technology c. No, a professional teacher, regardless of teaching
b. Offered as a separate subject experience, ought to go through CPE
c. Emphasize for creativity and productivity d. Yes, because he taught for thirty two years and may have
d. Integrated with technology and home economics mastered the traded
791. Which of the following is a mark of a good teacher? 800. Education is a continuous process of experiencing and visiting or
a. Has the habit of preparing for visual aids reorganizing experiences according to a Progressivist. What does
b. Has the mastery of the lesson it mean?
c. Has the control of the class a. Education takes place anytime and anywhere
d. Has the capability to implement corporal punishment b. Education goes on throughout life
792. The main function of a philosophy of education is to: c. Education happens formally or informally
a. Aid the learner to build his own personal philosophy d. Education begins and ends in school
b. Define the goals and set the direction for which 801. The main contribution of the Arroyo Administration in education is
education is to strive Republic Act No. 9155. This provision:
c. Provide the academic background prerequisite to learning a. Started the national scholarship program
d. Reconsider existing education goals in the light of society’s b. Renamed the DECS to DepEd
needs c. Conceptualized the K-12 Education Program
793. The control and the administration of all educational institutions d. Established the study now-pay later system
shall be vested in the citizens of the Philippines is stipulated in: 802. The main Purpose of compulsory education of the constitution is
a. P.D.1006 c. 1987 Constitution to:
b. P.D. 6-A d. P.D. 176 a. Prepare students to be lawyer
794. During the class reunion of teacher, Eric learned that most of his b. Develop students with into responsible thinking citizen
classmates are successful in their fields. Also, he found out that c. Acquaint students with the historical development of the
most of them are wealthy because they have chosen a lucrative constitution
profession. Confronted with this situation, how should teacher Eric d. Make student’s constitutional expert
react? 803. Which of the following situations which violate the principle of
a. Hide to those asked his real profession respect?
b. Tell with pride that he is a teacher by profession a. Teacher B is giving special favor to students to please so that
c. Leave the event so as to avoid being asked about his she can get a remarkable result in the evaluation
profession b. Teacher A tells her student that what teacher B taught is
d. Answer not their question concerning his profession wrong
795. Teachers being the trustee of the cultural and educational heritage c. Teacher B, upon learning what teacher A did, asked the
of the nation are under obligation to: students not to attend her class
a. Promote obedience to the laws of the state d. All of the above
b. Promote national pride, cultivate love of country & instill 804. If a teacher states that specialization knows more and more about
allegiance to the constitution less and less, hence it is better to be generalist. What kind of
c. Transmit to learners such heritage and elevate national philosophy does he uphold?
morality a. Positivism c. Progressivism
d. All of the above b. Essentialism d. Existentialism
796. The principal is very much interested in a quality professional 805. Teacher Paul is an inspiration to almost all of the students. His
development program of her teachers. Which of the following efficiency and effectiveness in the profession is truly outstanding.
should she consider to realize this? Which of the following describes this attitude towards him?
a. Prescribe by top educational teachers a. Naturalism c. Idealism
b. Required for renewal of professional license b. Progressivism d. Perennialism
c. Responsive to identified teachers needs 806. Teacher finds teaching in a multi cultural classes very challenging.
d. Dependent on the availability of funds Which among the following choices will alleviate the difficulty of
797. Ms. Soriano wants to help in ending Ms. Del Mundo’s act of addressing these challenges?
immorality but doesn’t have to encourage confronting her. What a. She must embody a curriculum rather than perspective
she did was to write and secretly distribute copies of anonymous b. She must nurture diversity rather than practicing
letter to her colleagues. What should have been done instead? domination and oppression
a. Secretly give anonymous letter only to the people concerned c. She must consider stereo typing rather than cultural identities
b. If the charge is valid; present such charge under oath and biased attitudes
before her school head
39
d. She must welcome one sided view rather than the recognition a. Valium c. Retalin
of biases b. Haldol d. Thorazine
807. Ms. Janina is the most admired pre-school teacher in her school. 820. What is the main reason why children with ADHD have limited
Which among the following can best explain her being a good learning skills?
teacher? a. Act on impulse and cannot concentrate
a. She endorses all the projects of the school for her students b. Must take stimulants which shorten attention span
b. She manages to instill control to her students c. Are given sedatives which make them listen
c. She gives easy exams to her students d. Are mildly retarded
d. She adheres to the want of the parents for their children 821. Many concerned parents commonly make the mistake of:
808. Teacher Vincent bought a hamster in the class during the lecture a. Deliberately creating high levels of stress for their child
about mammals. The hamster is a device commonly known as a b. Unintentionally rewarding their children for creating stress
REALIA. Teacher can bring realia only when: c. Attempting to protect their children from all stress
a. Workable c. Feasible d. Unintentionally creating high levels of stress for their child
b. Available d. Affordable 822. Stuttering is commonly caused by:
809. When asked, students of teacher Eric described him as someone a. Problem with physical origin
who knows what he is talking about. Teacher Eric therefore b. Psychological consequence of permissive parenting
exhibits a power known as: c. Side effects of authoritarian parenting
a. Expert power c. Referent power d. Result of fixation caused by conflicts during toilet training
b. Reward power d. Legitimate power 823. How can parents foster initiative and independence in children?
810. When choosing an instructional aide or device, the primary a. Mastering psychomotor skills
consideration of the teacher would be: b. Encouragement from parents when a child plans carries out a
a. Suitability c. Availability task
b. Cost d. Efficiency c. Identity versus role confusion
811. To show disapproval to the misbehavior of the student, Teacher d. None of the above
Paul clears his throat and looks intently at the erring. This 824. Which stage considers teachers, peers and adults outside the
classroom management style is commonly known as: home important in shaping attitude toward oneself?
a. Direct appeal c. Dropping of name a. Initiative versus guilt
b. Proximity control d. Signal interference b. Industry versus inferiority
812. Which of the computer-based instructional tool can help you revise c. Trust versus mistrust
written work such as short stories and essays? d. Integrity versus despair
a. Spreadsheets c. Desktop publishin 825. When a child manifest mutism, self-destructive behavior and
b. Database d. Word processing echolalia, the child might be showing the symptoms of:
813. Ms. Agatep wants to show to her class a magnified picture of the a. Anorexia nervosa c. Dyslexia
Mt. Pinatubo’s crater fixed on a bond paper. Which of the following b. Childhood autism d. ADHD
tools can she use? 826. When a person’s moral choices are determined by the direct
a. Slides c. Overhead Projector consequences of actions. He is most like in the stage of:
b. Filmstrip d. Opaque Projector a. Conventional c. Post conventional
814. As a teacher employing the project-based multimedia learning b. Concrete d. Pre conventional
(PBML) strategy, what are some limitations teachers expect from 827. What is the motivation of the person who paints for the sheer
the encounter? enjoyment of creating artwork?
I. There is a need for extending the to use several media a. Insufficient c. Extrinsic
II. The presentation of the product is not an easy task b. Intrinsic d. Intrinsic & Extrinsic
III. The technology skills to produce a product maybe lacking 828. Providing variety of learning activities to students is a
a. I, II, III c. I only characteristics of a teacher who understands the principle of:
b. III only d. II only a. Proactive teaching as a modem technique of teaching
815. B.F. skinner is a known psychologist and the one who first to b. Facilitating learning with emphasis on individual
describe operant conditioning. Which of the techniques is an differences
application of operant conditioning? c. Reward as a potential means of increasing the participation
a. Master learning d. Allowing the student to be exposed to various teaching
b. Process approach techniques
c. Project method 829. All of the following shows respect for individual differences except:
d. Computer assisted instruction a. Give greater attention to gifted learners
816. Which of the following choices is considered as social force that b. Treat all learners alike while the classroom
affects the school and the curriculum? c. Provide for a variety of learning activities
a. Nature of knowledge d. Prepare modules for slow learners in class
b. Learners’ characteristics 830. Students who are disobedient and display negative attitudes
c. Learner’s style towards others are best handled by teacher who will:
d. Changes in gender roles a. Detain him after office hours for him to do what he has been
817. Which of the following is an expression of child’s interest in his ordered to
body? b. Avoid giving him orders or if you do and the objects take back
a. Comparing themselves with others the order
b. Looking at themselves in the mirror c. Take every opportunity to praise him for every positive
c. Commenting on various parts o the body attitude displays
d. Looking the picture of adult men and women d. Insist on compliance to the same degree required of pupils
818. Motor development is satisfied by a particular child who; 831. Which of the following develops critical thinking skills among the
a. Recognize the different sizes of toys given to him students?
b. Learns how to walk, run, steer and jump a. Asking convergent question
c. Has playmates within the neighborhood & is popular among b. Blind obedience to authority
kids c. A willingness to suspend judgment
d. Known how to control his emotions because he could not ride d. Asking low level questions
the motor cycle 832. A child who always fights with his/her classmates, who has a very
819. Which among the following drugs is commonly used for children short attention span, and who has frequent tantrums is believed to
with ADHD? be suffering from:
40
a. Mental retardation c. Attending Exhibit
b. Attention deficit hyperactivity disorder d. Hearing and listening
c. Down syndrome 846. This is pre-planned collection of sample of student works,
d. Learning disability assessed results and other output produced by the students:
833. Teacher Elaine has been lecturing for more than an hour and she a. Anecdotal report c. Portfolio
notice that students are not anymore able to absorb additional b. Diary d. Observation image
information. This phenomenon is known as: 847. Which of the following statements is one of the strength of an
a. Stagnation c. Boredom autobiography as a technique for personality appraisal?
b. Procrastination d. Plateau of learning a. It can replace data obtain from other data techniques
834. Planned ignoring, signal interference and proximity control are b. It may be read by unauthorized people
techniques used in: c. It gives complete data about the author
a. Operant conditioning d. It makes possible presentation of intimate experiences
b. Managing surface behavior 848. The present Philippine Teachers professionalization Act had its
c. Managing temper tantrums beginning on what period of roman history?
d. Life space interviewing a. 295 B.C. - schools were elementary only
835. A foreigner who is studying here in the Philippines was turned off b. A.D. 100 - A.D. 175 - government increased its subsidy
by the Filipinos way of eating Balut and Frogs: for education
a. Xenocentrism c. Colonial mentality c. 132 B.C. - A.D. 100 - Latin literature and grammar were
b. Ethnocentrismd d. Culture shock perfected
836. Who among the following claimed that children are natural learners d. A.D. 274-259 - government establish a monopoly on
and therefore must be taught in natural settings? education
a. Kohlberg c. Montessori 849. Whose philosophy influences the present emphasis on character
b. Piaget d. Froebel education and values education in our school system?
837. The concept that learning to read or write does not happen quickly a. Tagore c. Confucius
but is build upon many small steps that occur over the course of b. Gandhi d. Bonifacio
the child’s early childhood. 850. During the distribution of the report card, which of the following
a. Innate literacy must be the foremost concern of teacher?
b. Emergent literacy a. Discuss the projects of the school
c. Functional literacy b. Discuss the unsettled bill of the students
d. Academic Literacy c. Discuss the progress as well as the deficiencies of the
838. A boy is closer to his mother and a girl is close to her father. These stu7dent
instances are under: d. Discuss the complaints of other teachers and classmates
a. Latent stage c. Phallic stage of the students
b. Oedipal Complex d. Pre-genital stage 851. After listening to the homily of the priest about fidelity, Catherine
839. Laughing at two year old child who uttered a bad word is not a has a moment of reflection. Her understanding of the value of
proper thing to do because in this stage of the child’s life, the child fidelity has become deeper as she related this to her past
is: experience. This typifies what kind of philosophy?
a. Considering the views of others a. Constructivism c. Humanism
b. Distinguishing right from wrong b. Reconstructivism d. Existentialism
c. Distinguishing sex differences 852. Teacher Vincent is a new teacher. He realizes that handling his
d. Socializing student’s misbehavior is a very demanding aspect of classroom
840. The school director emphasizes the necessity of clean and green management. In this regard he thought of giving up teaching.
environment to contribute to effective teaching and learning. This What advice can you give him?
is an example of: a. Report every student’s misbehavior to the principal
a. Establish rapport between teachers and pupils b. Agree with the class on what rules to follow
b. Providing an atmosphere conductive to learning c. Set the ground rules for the whole class
c. Providing adequate physical facilities d. None of the above
d. Utilize educational technology 853. Which of the following is a result of compressing a file?
841. The teachers are facilitators of learning. Which of the following a. The file size is smaller
negates this principle? b. The file deleted
a. Performs not only classrooms roles but community c. The three character extension is hidden
involvement as well d. All file properties
b. Focus background of the students 854. Mrs. Anita Kusing was not accepted by a certain company
c. Familiar with the latest innovations in teaching because of her age. This discrimination based on age is called
d. Uses pro-active discipline method a. Gerontism c. Agelism
842. Which of these philosophies is reflective of the Dewey’s which b. Autism d. Senilism
s\tresses the development of an individual capable of reflecting 855. Vincent parents do not want their child with ADHD to undergo
thinking specially that of being able to solve the problem he faces drug treatment, their better alternative would be
individually or collectively? a. Behavior modification or behavior management
a. Disciplinism c. Experimentation b. Psychotherapy
b. Developmentalism d. Rationalism c. Punishment
843. The present military training in our school curriculum is an d. Progressive inhibition
influence of: 856. Ripple effect can also be seen in misdemeanor. The teacher
a. Sparta c. Greece should therefore:
b. Rome d. Athens a. Reinforce positive behavior
844. Basic education includes secondary education. Which of the b. Immediately responds to misbehavior
following contributed to the establishment of secondary schools? c. Be consistent in classroom management
a. Reformation c. Realistic d. Count 1 to 10 before she gets angry
b. Rationalism d. Human Education 857. Metacognition is primarily characterized by:
845. The nearest to the real thing according to Edgar Dale’s Cone of a. Recalling the past lesson
Experience is: b. Thinking about their thinking
a. Watching demo c. Visualizing in advance
b. Viewing images d. Formulating hypothesis
41
858. A person who is friendly and has a capacity to make people d. Standards, learning competencies, and evaluation
laugh possesses: 870. The following are examples of learning centered design except:
a. Interpersonal intelligence a. Child centered design
b. Naturalistic intelligence b. Life situation design
c. Spatial intelligence c. Humanistic centered design
d. Intrapersonal intelligence d. Experience centered design
859. Which of the following is an example of norm-referenced 871. Which of the following statements about computer viruses are
interpretation? TRUE?
a. Josh’s test score is higher that 89% of the class a. Text files are the only files to be corrupted by virus.
b. Francis set up his laboratory equipment in 2 minutes b. Files damaged by computer viruses can be cured
c. RJ solve five problems correctly out of thirty words c. Files are always permanently damage by viruses
d. Bea must spell twenty five words correctly out of thirty d. Compressed files can never be damaged by viruses
words 872. The students of Teacher Corazon feels that their teacher has an
860. A type of error committed in grading the performance of the “eye behind her head” this characteristics of the teacher is
students by the rater who avoids both extremes of the scale and known as:
tends to rate everyone as revenge a. Referent Power c. Pygmalion Effect
a. Severity error b. Withitness d. Rosenthal Effect
b. Central tendency error 873. After studying the Principle of identity, teaching Beng ask her
c. Generosity error students to determine which among the given set of problems
d. Logical error conforms to the said identity. This shows that teacher beng
861. Which of the following test items can be effectively measure upholds this kind of philosophy?
higher order of cognitive learning objectives? a. Perennialism c. Essentialism
a. Achievement test b. Progressivism d. Naturalism
b. Extended essay test 874. How will you classify the purpose of the school as it concerns
c. Completion test with the training and preparation of citizens for the world of
d. Objective test work?
862. A student finding it hard to read. When the guidance counselor a. Political Purpose c. Economic Purpose
traced the child’s history, the counselor was able to find out the b. Cognitive Purpose d. Social Purpose
student came from a dysfunctional family, aside from that the 875. A child submitted a poor written report but package with boringly
child was abused and neglected. What could have caused the colored paper cover. This showcase____.
students reading disability? a. Art over academics
a. Poor teaching b. “porma” over substance
b. Emotional factors c. Substance over “porma”
c. Neurological factors d. Art over science
d. Immunity 876. Clearance has inherent skills in taking care of plants. It is highly
863. The best example Operant conditioning among the following is: possible that she has ________ intelligence.
a. Fostering conductive learning environment a. Intrapersonal intelligence
b. Connecting facts and concept b. Naturalistic intelligence
c. Using reinforcement c. Spatial intelligence
d. Using manipulative device d. Existential intelligence
864. The failure in the test of independence among Filipino students 877. According to Erickson, a child who is cold towards that people
can be attributed to among him might have failed to attain what basic goal based on
a. Lack of skills psychosocial development?
b. High degree of dependence to authority a. Autonomy c. Initiative
c. Strong family ties b. Trust d. Mistrust
d. All of the above 878. What is most likely characteristic of children aged 3 to 5
865. Mrs. Soriano is admitted for being an effective classroom according to Erickson?
manager. She is not only friendly but at the same time a. Mischievous c. Lazy
be_______ b. Egocentric d. Altruistic
a. Confident c. Analytical 879. Mrs. Cruz dreams to organize a seminar with a known poet from
b. Business-like d. Buddy-buddy another country but she cannot afford to spend for a
866. Which of the following would be the most fitting action for the transportation of the said poet. Which of the following can she
teacher who is having a relationship with his/her student? use so she won’t need to spend much?
a. Deny the relationship a. Distant communication
b. Continue the relationship and exercise utmost b. Instant messaging
professional discretion about this c. Video conferencing
c. Enjoy the relationship while it last d. Podcasting
d. Defer the relationship until they are already to admit it 880. Teacher Lally wants her students to express their opinions
867. The following is TRUE in the development of understanding in regarding a certain government issue. Which of the following
early childhood, except: can she use to do this?
a. Abilities to reason and to see relationship a. Forum c. Email
b. Sensory experiences b. Blog d. Group messaging system
c. Ability to explore their environment 881. Which is the first step in planning periodic test?
d. Ability to ask questions a. Select the type of test to be used
868. Teacher Eric would like to compare and contrast plant cell vs. b. Construct a table specification
animal cell. He would most likely use: c. Go back to the instructional objectives
a. Fishbone diagram c. Tree Diagram d. Determine the group of whom the test is intended
b. Venn Diagram d. KWL chart 882. Below is a list of methods used to establish the reliability of a
869. Which of the following are the basic components of curriculum test, which method is questionable due to practice and
design? familiarity?
a. Assessment, teaching strategies and textbooks a. Split half c. Kuder Richardson
b. Content, structure and deliver b. Equivalent form d. Test retest
c. Philosophy, goals and objectives 883. What is meaning of TOS in the parlance of test construction?
42
a. Table of specifics 895. Teacher Anna set 85% accuracy in a test on predicting the kind
b. Table of specification of weather given 5 different atmospheric conditions. May obtain
c. Table of specific test a score of 82% can be interpreted as:
d. Table of subject a. She is 3% short of the set percentile score
884. Ms. Soriano will be absent for two days because of a national b. She did not meet the set standards by 3%
conference. She wants her student to work on a certain module. c. She obtained 82% percentile score
What is the fastest way of sending the module to her students d. She is higher than 82% of the group
while she is away? 896. Teacher Vincent wants to establish the reality of test in biology.
a. Share it to all her students in her live account Which of the following will he accomplish?
b. Email a module to her students I. Administer a parallel test
c. Send the module through group instant messaging
d. Message each of the students on a social networking site II. Split the test
885. Teacher Vincent is assigned to handle a multiple grade class.
What instructional material must be used to provide the need of III. Construct a variety of items
each grade?
a. Differentiated materials to cater to different levels IV. Administer the same test twice
b. Multisensory materials
c. Colorful, useful and durable materials a. I, III, IV c. I, II
d. Materials of high level thinking skills b. I, II, IV d. I, IV
886. A document publishes by a school district that identifies rules of 897. Teacher Vilma constructed a matching type test item. IN her
behavior that must be followed by anyone using the school column of descriptions are combinations presidents, current
districts computers, network, or internet connection. issues, and sports. Which of the constructing a matching type
a. Ethical internet use policy test items was NOT followed?
b. Classroom rules a. The descriptions must be homogenous
c. Acceptable use policy b. The options must be greater than the descriptions
d. Fair use c. The descriptions must be heterogeneous
887. Which of the following are the rationales behind using d. Arrange the options according the orders
technology in the classrooms? 898. Which of the following does not belong to the group when we
I. Motivational talk about projective personality test?
II. Unique instructional capabilities a. Sentence completion test
III. Increase Teachers’ Productivity b. Word association test
a. I only c. II and III only c. Interview
b. I and II only d. I, II, and III d. Thematic Apperception test
888. Using Bloom’s Taxonomy, the highest among the following is: 899. Which of the following is the main purpose of administering a
a. Critical Comprehension pre test and post test to the stu8dents?
b. Critical evaluation a. Measure the value of the materials used
c. Integration b. Accustom the students in frequent testing
d. Literal comprehension c. Measure gain in learning
889. Among the following educators, who proposed the placement of d. Measure the effectiveness of instruction
children in a “prepared environment”? 900. Which is implied by a positively skewed score distribution?
a. Thorndike c. Kilpatrick a. The mean, The median and the moderate are equal
b. Montessori d. Froebel b. Most of the scores are low
890. To improve comprehension and retention among the students, c. Most of the scores are high
the teachers best option would be use d. The mode is high
a. Six thing hats
b. SQ3R
c. Contextual Clues
d. Autodidactic Activities
891. Who among the philosophers considered habits and reasons as
equally important Forces to be cultivated in education?
a. Avicenna c. Jerome Bruner
b. Aristotle d. Maria Montessori
892. What practice(s) will demonstrate the teachers genuine concern
on the learning of students?
a. Confer progress of students to their parent
b. Guides students to meet their learning goals
c. Validates if learning goals were met
d. All of the above
893. Which of the following is the most important component of
educational reform?
a. Allowing the students to participate more in extra-curricular
activities
b. Implementing a better curriculum for students
c. Hiring only the excellent teachers
d. Involving parents in the dedication for the children
894. To increase the difficulty of a multiple choice test item, which of
the following should be done?
a. Make the options equal in length
b. Make the options homogeneous
c. Make it grammatically correct
d. Make the stem

43
44

You might also like